CAT  >  CAT Mock Test Series  >  CAT Practice Test - 17 Download as PDF

CAT Practice Test - 17 - CAT


Test Description

100 Questions MCQ Test CAT Mock Test Series - CAT Practice Test - 17

CAT Practice Test - 17 for CAT 2023 is part of CAT Mock Test Series preparation. The CAT Practice Test - 17 questions and answers have been prepared according to the CAT exam syllabus.The CAT Practice Test - 17 MCQs are made for CAT 2023 Exam. Find important definitions, questions, notes, meanings, examples, exercises, MCQs and online tests for CAT Practice Test - 17 below.
Solutions of CAT Practice Test - 17 questions in English are available as part of our CAT Mock Test Series for CAT & CAT Practice Test - 17 solutions in Hindi for CAT Mock Test Series course. Download more important topics, notes, lectures and mock test series for CAT Exam by signing up for free. Attempt CAT Practice Test - 17 | 100 questions in 180 minutes | Mock test for CAT preparation | Free important questions MCQ to study CAT Mock Test Series for CAT Exam | Download free PDF with solutions
1 Crore+ students have signed up on EduRev. Have you? Download the App
CAT Practice Test - 17 - Question 1

The passage given below is followed by a question. Choose the most appropriate answer to each question.

Today we are faced with the harsh reality that the treatment or prevention of infectious diseases has not made quantum advances since the early successes of vaccines and antimicrobial therapies. In a sense, the world is headed backward, as once-treatable microbes become resistant to existing therapies, and new infections for which there are no effective interventions continue to arise. This situation represents a serious and imminent threat to the world.

The emergence of a highly lethal and rapidly spreading antimicrobial- resistant infection would lead to untold numbers of deaths and unimaginable misery. The consequences could be similar in magnitude to a large-scale terrorist attack. Communities could be walled off, national borders closed, and travel could be restricted or even suspended. Health systems could disintegrate or collapse, as could economies. The possibility of such an apocalyptic scenario suggests that the threat of infectious diseases is among the most important challenges that humankind faces. It is not just a public health risk; it is a threat to national and global security. Thus, it must be met with a comprehensive and effective solution.

The research and development required to produce new medicines or vaccines is time-consuming, often taking more than a dozen years. It is also very expensive, costing hundreds of millions of dollars for every new product. Moreover, there is no guarantee of success; indeed, for each successful product, there are as many as nine equally promising candidates that fail. Given the risks involved, it is not surprising that pharmaceutical companies are very careful in their choice of investments in new drug or vaccine programs, selecting only those that promise financial gains sufficient to cover the costs of both successes and failures and provide a reasonable return on the required investment.

Almost every country is prepared to channel a large percentage of its GDP toward investments in national defense or security. The global threat of emerging or resistant infections must be viewed first and foremost in that context, with all countries committed to providing financing, intellectual capital, and available resources to support the discovery, development, manufacture, stockpiling, and equitable distribution of new antimicrobial agents and vaccines. Unless countries recognize the risks they face, they are unlikely to make such a commitment. It goes without saying that this would be a complicated undertaking, with many details to be worked out. But somehow we must suspend disbelief and take action now, lest we be caught off-guard against an imminent global threat. This is a battle we cannot afford to lose.

 

Q.According to the passage, which of the following is likely to pose a serious and imminent threat to the world? 

Detailed Solution for CAT Practice Test - 17 - Question 1

The passage talks about the emergence of highly lethal and rapidly spreading drug-resistant microbes and new infections, this validates option 3.
Option 1 can be ruled out as the consequences of the spread of antimicrobial-resistant infections have been compared to a terrorist attack.
Option 2 is a point mentioned in the passage, but it does not represent a serious threat to the world.
Option 4 cannot be corroborated from the passage.
Hence, the correct answer is option 3.

CAT Practice Test - 17 - Question 2

The passage given below is followed by a question. Choose the most appropriate answer to each question.

Today we are faced with the harsh reality that the treatment or prevention of infectious diseases has not made quantum advances since the early successes of vaccines and antimicrobial therapies. In a sense, the world is headed backward, as once-treatable microbes become resistant to existing therapies, and new infections for which there are no effective interventions continue to arise. This situation represents a serious and imminent threat to the world.

The emergence of a highly lethal and rapidly spreading antimicrobial- resistant infection would lead to untold numbers of deaths and unimaginable misery. The consequences could be similar in magnitude to a large-scale terrorist attack. Communities could be walled off, national borders closed, and travel could be restricted or even suspended. Health systems could disintegrate or collapse, as could economies. The possibility of such an apocalyptic scenario suggests that the threat of infectious diseases is among the most important challenges that humankind faces. It is not just a public health risk; it is a threat to national and global security. Thus, it must be met with a comprehensive and effective solution.

The research and development required to produce new medicines or vaccines is time-consuming, often taking more than a dozen years. It is also very expensive, costing hundreds of millions of dollars for every new product. Moreover, there is no guarantee of success; indeed, for each successful product, there are as many as nine equally promising candidates that fail. Given the risks involved, it is not surprising that pharmaceutical companies are very careful in their choice of investments in new drug or vaccine programs, selecting only those that promise financial gains sufficient to cover the costs of both successes and failures and provide a reasonable return on the required investment.

Almost every country is prepared to channel a large percentage of its GDP toward investments in national defense or security. The global threat of emerging or resistant infections must be viewed first and foremost in that context, with all countries committed to providing financing, intellectual capital, and available resources to support the discovery, development, manufacture, stockpiling, and equitable distribution of new antimicrobial agents and vaccines. Unless countries recognize the risks they face, they are unlikely to make such a commitment. It goes without saying that this would be a complicated undertaking, with many details to be worked out. But somehow we must suspend disbelief and take action now, lest we be caught off-guard against an imminent global threat. This is a battle we cannot afford to lose.

 

Q.The following could be the repercussions of the spread of infectious diseases, except: 

Detailed Solution for CAT Practice Test - 17 - Question 2

The passage states “Communities could be walled off, national borders closed, and travel could be restricted or even suspended. Health systems could disintegrate or collapse, as could economies.” This validates options 1,2 and 3. Option 4 is not validated from any data in the passage.
Hence, the correct answer is option 4.

CAT Practice Test - 17 - Question 3

The passage given below is followed by a question. Choose the most appropriate answer to each question.

Today we are faced with the harsh reality that the treatment or prevention of infectious diseases has not made quantum advances since the early successes of vaccines and antimicrobial therapies. In a sense, the world is headed backward, as once-treatable microbes become resistant to existing therapies, and new infections for which there are no effective interventions continue to arise. This situation represents a serious and imminent threat to the world.

The emergence of a highly lethal and rapidly spreading antimicrobial- resistant infection would lead to untold numbers of deaths and unimaginable misery. The consequences could be similar in magnitude to a large-scale terrorist attack. Communities could be walled off, national borders closed, and travel could be restricted or even suspended. Health systems could disintegrate or collapse, as could economies. The possibility of such an apocalyptic scenario suggests that the threat of infectious diseases is among the most important challenges that humankind faces. It is not just a public health risk; it is a threat to national and global security. Thus, it must be met with a comprehensive and effective solution.

The research and development required to produce new medicines or vaccines is time-consuming, often taking more than a dozen years. It is also very expensive, costing hundreds of millions of dollars for every new product. Moreover, there is no guarantee of success; indeed, for each successful product, there are as many as nine equally promising candidates that fail. Given the risks involved, it is not surprising that pharmaceutical companies are very careful in their choice of investments in new drug or vaccine programs, selecting only those that promise financial gains sufficient to cover the costs of both successes and failures and provide a reasonable return on the required investment.

Almost every country is prepared to channel a large percentage of its GDP toward investments in national defense or security. The global threat of emerging or resistant infections must be viewed first and foremost in that context, with all countries committed to providing financing, intellectual capital, and available resources to support the discovery, development, manufacture, stockpiling, and equitable distribution of new antimicrobial agents and vaccines. Unless countries recognize the risks they face, they are unlikely to make such a commitment. It goes without saying that this would be a complicated undertaking, with many details to be worked out. But somehow we must suspend disbelief and take action now, lest we be caught off-guard against an imminent global threat. This is a battle we cannot afford to lose.

 

Q.In the passage, which of the following is/are obstacles faced by pharmaceuticals?
A. Bottlenecks in development and production of new medicines and vaccines

B. Constraints in raising capital for research and development

C. Availability of skilled workers and competent scientists to develop new medicines and vaccines 

Detailed Solution for CAT Practice Test - 17 - Question 3

The passage states “The research and development required to produce new medicines or vaccines is time-consuming, often taking more than a dozen years.” indicating statement A as the correct answer.
Statements B and C cannot be inferred from the passage. Hence, the correct answer is option 1.

CAT Practice Test - 17 - Question 4

The passage given below is followed by a question. Choose the most appropriate answer to each question.

Today we are faced with the harsh reality that the treatment or prevention of infectious diseases has not made quantum advances since the early successes of vaccines and antimicrobial therapies. In a sense, the world is headed backward, as once-treatable microbes become resistant to existing therapies, and new infections for which there are no effective interventions continue to arise. This situation represents a serious and imminent threat to the world.

The emergence of a highly lethal and rapidly spreading antimicrobial- resistant infection would lead to untold numbers of deaths and unimaginable misery. The consequences could be similar in magnitude to a large-scale terrorist attack. Communities could be walled off, national borders closed, and travel could be restricted or even suspended. Health systems could disintegrate or collapse, as could economies. The possibility of such an apocalyptic scenario suggests that the threat of infectious diseases is among the most important challenges that humankind faces. It is not just a public health risk; it is a threat to national and global security. Thus, it must be met with a comprehensive and effective solution.

The research and development required to produce new medicines or vaccines is time-consuming, often taking more than a dozen years. It is also very expensive, costing hundreds of millions of dollars for every new product. Moreover, there is no guarantee of success; indeed, for each successful product, there are as many as nine equally promising candidates that fail. Given the risks involved, it is not surprising that pharmaceutical companies are very careful in their choice of investments in new drug or vaccine programs, selecting only those that promise financial gains sufficient to cover the costs of both successes and failures and provide a reasonable return on the required investment.

Almost every country is prepared to channel a large percentage of its GDP toward investments in national defense or security. The global threat of emerging or resistant infections must be viewed first and foremost in that context, with all countries committed to providing financing, intellectual capital, and available resources to support the discovery, development, manufacture, stockpiling, and equitable distribution of new antimicrobial agents and vaccines. Unless countries recognize the risks they face, they are unlikely to make such a commitment. It goes without saying that this would be a complicated undertaking, with many details to be worked out. But somehow we must suspend disbelief and take action now, lest we be caught off-guard against an imminent global threat. This is a battle we cannot afford to lose.

 

Q.Which of the following cannot be inferred from the passage?

Detailed Solution for CAT Practice Test - 17 - Question 4

Options 1 and 3 are supported by the last paragraph. Option 2 is supported by the penultimate paragraph. Option 4 is not entirely true, as the topic of provisions for incentives for drug development has not been mentioned in the passage. Hence, the correct answer is option 4.

CAT Practice Test - 17 - Question 5

The passage given below is followed by a question. Choose the most appropriate answer to each question.

Today we are faced with the harsh reality that the treatment or prevention of infectious diseases has not made quantum advances since the early successes of vaccines and antimicrobial therapies. In a sense, the world is headed backward, as once-treatable microbes become resistant to existing therapies, and new infections for which there are no effective interventions continue to arise. This situation represents a serious and imminent threat to the world.

The emergence of a highly lethal and rapidly spreading antimicrobial- resistant infection would lead to untold numbers of deaths and unimaginable misery. The consequences could be similar in magnitude to a large-scale terrorist attack. Communities could be walled off, national borders closed, and travel could be restricted or even suspended. Health systems could disintegrate or collapse, as could economies. The possibility of such an apocalyptic scenario suggests that the threat of infectious diseases is among the most important challenges that humankind faces. It is not just a public health risk; it is a threat to national and global security. Thus, it must be met with a comprehensive and effective solution.

The research and development required to produce new medicines or vaccines is time-consuming, often taking more than a dozen years. It is also very expensive, costing hundreds of millions of dollars for every new product. Moreover, there is no guarantee of success; indeed, for each successful product, there are as many as nine equally promising candidates that fail. Given the risks involved, it is not surprising that pharmaceutical companies are very careful in their choice of investments in new drug or vaccine programs, selecting only those that promise financial gains sufficient to cover the costs of both successes and failures and provide a reasonable return on the required investment.

Almost every country is prepared to channel a large percentage of its GDP toward investments in national defense or security. The global threat of emerging or resistant infections must be viewed first and foremost in that context, with all countries committed to providing financing, intellectual capital, and available resources to support the discovery, development, manufacture, stockpiling, and equitable distribution of new antimicrobial agents and vaccines. Unless countries recognize the risks they face, they are unlikely to make such a commitment. It goes without saying that this would be a complicated undertaking, with many details to be worked out. But somehow we must suspend disbelief and take action now, lest we be caught off-guard against an imminent global threat. This is a battle we cannot afford to lose.

 

Q.“The possibility of such an apocalyptic scenario suggests that the threat of infectious diseases is among the most important challenges that humankind faces.” This implies: 

Detailed Solution for CAT Practice Test - 17 - Question 5

Option 1 is vague and not related to the matter in hand. So, eliminate option 1.
Option 2 is beyond the scope of the passage.
The sentence talks about the possibility of an event, in this case the threat of infectious diseases that could cause destruction or damage on a catastrophic scale. Taking this into consideration, option 3 is apt.
Option 4 is a probable conclusion and not an implication.
Hence, the correct answer is option 3.

CAT Practice Test - 17 - Question 6

The passage given below is followed by a question. Choose the most appropriate answer to each question.

Today we are faced with the harsh reality that the treatment or prevention of infectious diseases has not made quantum advances since the early successes of vaccines and antimicrobial therapies. In a sense, the world is headed backward, as once-treatable microbes become resistant to existing therapies, and new infections for which there are no effective interventions continue to arise. This situation represents a serious and imminent threat to the world.

The emergence of a highly lethal and rapidly spreading antimicrobial- resistant infection would lead to untold numbers of deaths and unimaginable misery. The consequences could be similar in magnitude to a large-scale terrorist attack. Communities could be walled off, national borders closed, and travel could be restricted or even suspended. Health systems could disintegrate or collapse, as could economies. The possibility of such an apocalyptic scenario suggests that the threat of infectious diseases is among the most important challenges that humankind faces. It is not just a public health risk; it is a threat to national and global security. Thus, it must be met with a comprehensive and effective solution.

The research and development required to produce new medicines or vaccines is time-consuming, often taking more than a dozen years. It is also very expensive, costing hundreds of millions of dollars for every new product. Moreover, there is no guarantee of success; indeed, for each successful product, there are as many as nine equally promising candidates that fail. Given the risks involved, it is not surprising that pharmaceutical companies are very careful in their choice of investments in new drug or vaccine programs, selecting only those that promise financial gains sufficient to cover the costs of both successes and failures and provide a reasonable return on the required investment.

Almost every country is prepared to channel a large percentage of its GDP toward investments in national defense or security. The global threat of emerging or resistant infections must be viewed first and foremost in that context, with all countries committed to providing financing, intellectual capital, and available resources to support the discovery, development, manufacture, stockpiling, and equitable distribution of new antimicrobial agents and vaccines. Unless countries recognize the risks they face, they are unlikely to make such a commitment. It goes without saying that this would be a complicated undertaking, with many details to be worked out. But somehow we must suspend disbelief and take action now, lest we be caught off-guard against an imminent global threat. This is a battle we cannot afford to lose.

 

Q.What is the passage about?

Detailed Solution for CAT Practice Test - 17 - Question 6

Measures to overcome the threat of resistant microbes is not mentioned in the passage as it only tries to throw light on the seriousness of the issue. Thus, option 1 can be ruled out.
The passage focuses on how antimicrobial resistant microbes could be a threat to national and global security. Thus, option 2 is apt.
The passage does not divulge anything about the healthcare scenario, neither does it talk about upgrading or modernising pharmaceutical industry. Thus, options 3 and 4 can be ruled out. Hence, the correct answer is option 2

CAT Practice Test - 17 - Question 7

A passage is followed by questions pertaining to the passage. Read the passage and answer the questions. Choose the most appropriate answer.

Until about 1830, pizza was sold from open-air stands and out of pizza bakeries. Pizzerias keep this age-old tradition alive today. It is possible to enjoy pizza wrapped in paper and a drink sold from open-air stands outside the premises. Antica Pizzeria Port'Alba in Naples is widely regarded as the city's first pizzeria. It started producing pizzas for peddlers in 1738 but expanded to a pizza restaurant with chairs and tables in 1830. It still serves pizza from the same premises today.

A description of pizza in Naples around 1830 is given by the French writer and food expert Alexandre Dumas, pere in his work Le Corricolo, Chapter VIII. He writes that pizza was the only food of the humble people in Naples during winter and that "in Naples pizza is flavored with oil, lard, tallow, cheese, tomato, or anchovies."

The Neapolitans take their pizza very seriously. Purists, like the famous pizzeria “Da Michele” in Via C. Sersale (founded 1870), consider there to be only two true pizzas — the Marinara and the Margherita — and that is all they serve. These two "pure" pizzas are the ones preferred by many Italians today.
The Marinara is the older of the two and has a topping of tomato, oregano, garlic and extra virgin olive oil. It is named “Marinara” not because it has seafood on it (it doesn't) but because it was the food prepared by "la marinara", the seaman's wife, for her sea faring husband when he returned from fishing trips in the Bay of Naples.

 

Q.Why could the “Marinara” be easily mistaken as a seafood pizza?

Detailed Solution for CAT Practice Test - 17 - Question 7

The question at hand is not why the “Marinara” is named so, rather why its meaning could be confused with that of a seafood pizza.
Option 1 is the correct answer as its name has seafood connotations.
Option 3, which is the real reason behind calling it “Marinara”, does not answer the question.
Options 2 and 4 aren’t supported by the passage.
Hence, the correct answer is option 1.

CAT Practice Test - 17 - Question 8

A passage is followed by questions pertaining to the passage. Read the passage and answer the questions. Choose the most appropriate answer.

Until about 1830, pizza was sold from open-air stands and out of pizza bakeries. Pizzerias keep this age-old tradition alive today. It is possible to enjoy pizza wrapped in paper and a drink sold from open-air stands outside the premises. Antica Pizzeria Port'Alba in Naples is widely regarded as the city's first pizzeria. It started producing pizzas for peddlers in 1738 but expanded to a pizza restaurant with chairs and tables in 1830. It still serves pizza from the same premises today.

A description of pizza in Naples around 1830 is given by the French writer and food expert Alexandre Dumas, pere in his work Le Corricolo, Chapter VIII. He writes that pizza was the only food of the humble people in Naples during winter and that "in Naples pizza is flavored with oil, lard, tallow, cheese, tomato, or anchovies."

The Neapolitans take their pizza very seriously. Purists, like the famous pizzeria “Da Michele” in Via C. Sersale (founded 1870), consider there to be only two true pizzas — the Marinara and the Margherita — and that is all they serve. These two "pure" pizzas are the ones preferred by many Italians today.
The Marinara is the older of the two and has a topping of tomato, oregano, garlic and extra virgin olive oil. It is named “Marinara” not because it has seafood on it (it doesn't) but because it was the food prepared by "la marinara", the seaman's wife, for her sea faring husband when he returned from fishing trips in the Bay of Naples.

 

Q.The passage implies that the Margherita and the Marinara are the only two “pure” pizzas: 

Detailed Solution for CAT Practice Test - 17 - Question 8

The passage puts the word “pure” in quotes, because it is trying to imply that these pizzas are only considered pure by the purists, not that they actually are. One can also conclude that they were “probably” the first ones created, hence the purists’ loyalty.
Options 1 is not substantiated by the data in the passage, nor can it be inferred.
Option 2 provides the incorrect name and option 3, though true, is irrelevant to the question stem.
Hence, the correct answer is option 4.

CAT Practice Test - 17 - Question 9

A passage is followed by questions pertaining to the passage. Read the passage and answer the questions. Choose the most appropriate answer.

Until about 1830, pizza was sold from open-air stands and out of pizza bakeries. Pizzerias keep this age-old tradition alive today. It is possible to enjoy pizza wrapped in paper and a drink sold from open-air stands outside the premises. Antica Pizzeria Port'Alba in Naples is widely regarded as the city's first pizzeria. It started producing pizzas for peddlers in 1738 but expanded to a pizza restaurant with chairs and tables in 1830. It still serves pizza from the same premises today.

A description of pizza in Naples around 1830 is given by the French writer and food expert Alexandre Dumas, pere in his work Le Corricolo, Chapter VIII. He writes that pizza was the only food of the humble people in Naples during winter and that "in Naples pizza is flavored with oil, lard, tallow, cheese, tomato, or anchovies."

The Neapolitans take their pizza very seriously. Purists, like the famous pizzeria “Da Michele” in Via C. Sersale (founded 1870), consider there to be only two true pizzas — the Marinara and the Margherita — and that is all they serve. These two "pure" pizzas are the ones preferred by many Italians today.
The Marinara is the older of the two and has a topping of tomato, oregano, garlic and extra virgin olive oil. It is named “Marinara” not because it has seafood on it (it doesn't) but because it was the food prepared by "la marinara", the seaman's wife, for her sea faring husband when he returned from fishing trips in the Bay of Naples.

 

Q.Which of the following options could be a suitable title for this passage?

Detailed Solution for CAT Practice Test - 17 - Question 9

The passage clearly focuses on the history of pizza, and more so in Naples, Italy.
Since it doesn’t merely talk about how a pizza is made, but goes beyond to discuss its history, options 1 and 3 can be eliminated. Option 2 is very narrow and mentions only one aspect of the passage.
Hence, the correct answer is option 4.

CAT Practice Test - 17 - Question 10

The passage given below is followed by a question. Choose the most appropriate answer to each question.

Is anyone a born entrepreneur, a born leader or born to win? I believe everyone is born to win but you have to make winning a habit. Think of this: a quitter never wins, a winner never quits. I have been privileged to share the dream and journey of many entrepreneurs. A few traits are habits with all of them. A winner is a believer. Belief in oneself and one’s convictions — that inner voice of core strength to pursue dreams despite difficulties.

Along the way everyone they meet gets swept up in their dreams and aspirations. This belief is rooted in a bigger purpose than self- satisfaction. Winners are great listeners. They learn continuously; they take inputs; they don’t think they have all the answers; and they are confident to have the humility to hear other points of view.

Listening is about striving for comprehension and making oneself open to possibilities. Winners value self-development. They constantly challenge themselves and set aggressive goals and often surpass expectations. They make organisational learning a priority. Winners are unstoppable. They don’t count hurdles. They don’t brood about failures and risks. They are activators.

They have a clear compass on why they keep trying and what they set out to achieve. Hence they have the edge to get up, dust themselves off, and set out again and again.

Winners are decisive. They chart the course and make choices all the time while being transparent and suffused with a clarity of purpose. They are less prone to procrastination. They are resilient to change and very effective in communicating their decisions. Another word for this is nimbleness, and in these times, this quality is essential to innovate.
Is all this too much to ask? Of late, too often, I read are we expecting too much from young entrepreneurs. Was too much expected of Alexander when he became king at 20 and set out to conquer the world? Or of Akbar who inherited the empire at the age of 14? Being a founder, building a high-velocity organisation is no doubt high-pressure, but isn’t it a choice? Isn’t competing in the Olympics different from playing cricket in your backyard?
Over the years, I have a more nuanced take about winning itself.
Winning doesn’t always mean being first; winning means you are doing better than you have done before.

 


Q.Which of the following could be a suitable title for the above passage? 

Detailed Solution for CAT Practice Test - 17 - Question 10

The passage revolves around the endeavors of winners to perform better again and again which ultimately leads them on the path of success and differentiates them from the losers. This is captured best in option 3.
The entire focus here is on winning and the topic of losing has not been touched upon. Hence, option 1 is eliminated.
In the passage, the process of winning is the primary focus and not the benefits of winning. Thus, option 2 would be incorrect. The scope of the passage does not go beyond the process involved in winning and thus “winning isn’t everything” won’t be a suitable title here. Thus option 4 is eliminated.
Hence, the correct answer is option 3.

CAT Practice Test - 17 - Question 11

The passage given below is followed by a question. Choose the most appropriate answer to each question.

Is anyone a born entrepreneur, a born leader or born to win? I believe everyone is born to win but you have to make winning a habit. Think of this: a quitter never wins, a winner never quits. I have been privileged to share the dream and journey of many entrepreneurs. A few traits are habits with all of them. A winner is a believer. Belief in oneself and one’s convictions — that inner voice of core strength to pursue dreams despite difficulties.

Along the way everyone they meet gets swept up in their dreams and aspirations. This belief is rooted in a bigger purpose than self- satisfaction. Winners are great listeners. They learn continuously; they take inputs; they don’t think they have all the answers; and they are confident to have the humility to hear other points of view.

Listening is about striving for comprehension and making oneself open to possibilities. Winners value self-development. They constantly challenge themselves and set aggressive goals and often surpass expectations. They make organisational learning a priority. Winners are unstoppable. They don’t count hurdles. They don’t brood about failures and risks. They are activators.

They have a clear compass on why they keep trying and what they set out to achieve. Hence they have the edge to get up, dust themselves off, and set out again and again.

Winners are decisive. They chart the course and make choices all the time while being transparent and suffused with a clarity of purpose. They are less prone to procrastination. They are resilient to change and very effective in communicating their decisions. Another word for this is nimbleness, and in these times, this quality is essential to innovate.
Is all this too much to ask? Of late, too often, I read are we expecting too much from young entrepreneurs. Was too much expected of Alexander when he became king at 20 and set out to conquer the world? Or of Akbar who inherited the empire at the age of 14? Being a founder, building a high-velocity organisation is no doubt high-pressure, but isn’t it a choice? Isn’t competing in the Olympics different from playing cricket in your backyard?
Over the years, I have a more nuanced take about winning itself.
Winning doesn’t always mean being first; winning means you are doing better than you have done before.

 

Q.Which of the following, if true, weakens the argument in the above passage?
A. One has to be bom a winner and winning cannot be taught or learned.
B. Losers are no different than winners apart from the fact that they are short on luck.

Detailed Solution for CAT Practice Test - 17 - Question 11

The passage states that everyone is born to win but one has to make winning a habit. This is contradicted by statement A which weakens the main argument.
The passage talks extensively about various qualities that separate winners from the rest. Statement B is incorrect as luck has not been mentioned as a factor to differentiate winners from losers. Thus, statement B is weakening.
Hence, the correct answer is option 3.

CAT Practice Test - 17 - Question 12

The passage given below is followed by a question. Choose the most appropriate answer to each question.

Is anyone a born entrepreneur, a born leader or born to win? I believe everyone is born to win but you have to make winning a habit. Think of this: a quitter never wins, a winner never quits. I have been privileged to share the dream and journey of many entrepreneurs. A few traits are habits with all of them. A winner is a believer. Belief in oneself and one’s convictions — that inner voice of core strength to pursue dreams despite difficulties.

Along the way everyone they meet gets swept up in their dreams and aspirations. This belief is rooted in a bigger purpose than self- satisfaction. Winners are great listeners. They learn continuously; they take inputs; they don’t think they have all the answers; and they are confident to have the humility to hear other points of view.

Listening is about striving for comprehension and making oneself open to possibilities. Winners value self-development. They constantly challenge themselves and set aggressive goals and often surpass expectations. They make organisational learning a priority. Winners are unstoppable. They don’t count hurdles. They don’t brood about failures and risks. They are activators.

They have a clear compass on why they keep trying and what they set out to achieve. Hence they have the edge to get up, dust themselves off, and set out again and again.

Winners are decisive. They chart the course and make choices all the time while being transparent and suffused with a clarity of purpose. They are less prone to procrastination. They are resilient to change and very effective in communicating their decisions. Another word for this is nimbleness, and in these times, this quality is essential to innovate.
Is all this too much to ask? Of late, too often, I read are we expecting too much from young entrepreneurs. Was too much expected of Alexander when he became king at 20 and set out to conquer the world? Or of Akbar who inherited the empire at the age of 14? Being a founder, building a high-velocity organisation is no doubt high-pressure, but isn’t it a choice? Isn’t competing in the Olympics different from playing cricket in your backyard?
Over the years, I have a more nuanced take about winning itself.
Winning doesn’t always mean being first; winning means you are doing better than you have done before.

 

Q.Which of the following, if true, strengthens the argument in the above passage?
A. Thomas Edison, the famous scientist, invented the light bulb only after making 1,000 unsuccessful attempts.
B. Communication is not an identifying characteristic of a winner as there are a number of winners who have been poor communicators.

Detailed Solution for CAT Practice Test - 17 - Question 12

Statement A talks about Edison’s persistence despite repeated failures. This is directly in line with the passage which says “a quitter never wins, a winner never quits” and “winning means you are doing better than you have done before”. Thus, statement A is strengthening the argument in the passage.
The passage states that winners are effective communicators whereas statement B says the exact opposite. Thus, statement B is weakening the argument in the above passage.
Hence, the answer is option 1.

CAT Practice Test - 17 - Question 13

The passage given below is followed by a question. Choose the most appropriate answer to each question.

Is anyone a born entrepreneur, a born leader or born to win? I believe everyone is born to win but you have to make winning a habit. Think of this: a quitter never wins, a winner never quits. I have been privileged to share the dream and journey of many entrepreneurs. A few traits are habits with all of them. A winner is a believer. Belief in oneself and one’s convictions — that inner voice of core strength to pursue dreams despite difficulties.

Along the way everyone they meet gets swept up in their dreams and aspirations. This belief is rooted in a bigger purpose than self- satisfaction. Winners are great listeners. They learn continuously; they take inputs; they don’t think they have all the answers; and they are confident to have the humility to hear other points of view.

Listening is about striving for comprehension and making oneself open to possibilities. Winners value self-development. They constantly challenge themselves and set aggressive goals and often surpass expectations. They make organisational learning a priority. Winners are unstoppable. They don’t count hurdles. They don’t brood about failures and risks. They are activators.

They have a clear compass on why they keep trying and what they set out to achieve. Hence they have the edge to get up, dust themselves off, and set out again and again.

Winners are decisive. They chart the course and make choices all the time while being transparent and suffused with a clarity of purpose. They are less prone to procrastination. They are resilient to change and very effective in communicating their decisions. Another word for this is nimbleness, and in these times, this quality is essential to innovate.
Is all this too much to ask? Of late, too often, I read are we expecting too much from young entrepreneurs. Was too much expected of Alexander when he became king at 20 and set out to conquer the world? Or of Akbar who inherited the empire at the age of 14? Being a founder, building a high-velocity organisation is no doubt high-pressure, but isn’t it a choice? Isn’t competing in the Olympics different from playing cricket in your backyard?
Over the years, I have a more nuanced take about winning itself.
Winning doesn’t always mean being first; winning means you are doing better than you have done before.

 

Q.Which of the following agrees with the author’s perception of young entrepreneurs? 

Detailed Solution for CAT Practice Test - 17 - Question 13

The passage states with examples that high pressure and burden of expectations is a matter of choice and indicates that it may be a kind of ‘necessary evil’ and not something one should be complaining about. Thus, between options 1 and 2, option 2 would be a better answer.
The passage, in the last paragraph, is talking about young entrepreneurs facing high expectations. Thus, option 3 is eliminated.
The passage does not really state anything about the potential of seasoned or young entrepreneurs. Thus, option 4 is eliminated. Hence, the correct answer is option 2.

CAT Practice Test - 17 - Question 14

The passage given below is followed by a question. Choose the most appropriate answer to each question.

Is anyone a born entrepreneur, a born leader or born to win? I believe everyone is born to win but you have to make winning a habit. Think of this: a quitter never wins, a winner never quits. I have been privileged to share the dream and journey of many entrepreneurs. A few traits are habits with all of them. A winner is a believer. Belief in oneself and one’s convictions — that inner voice of core strength to pursue dreams despite difficulties.

Along the way everyone they meet gets swept up in their dreams and aspirations. This belief is rooted in a bigger purpose than self- satisfaction. Winners are great listeners. They learn continuously; they take inputs; they don’t think they have all the answers; and they are confident to have the humility to hear other points of view.

Listening is about striving for comprehension and making oneself open to possibilities. Winners value self-development. They constantly challenge themselves and set aggressive goals and often surpass expectations. They make organisational learning a priority. Winners are unstoppable. They don’t count hurdles. They don’t brood about failures and risks. They are activators.

They have a clear compass on why they keep trying and what they set out to achieve. Hence they have the edge to get up, dust themselves off, and set out again and again.

Winners are decisive. They chart the course and make choices all the time while being transparent and suffused with a clarity of purpose. They are less prone to procrastination. They are resilient to change and very effective in communicating their decisions. Another word for this is nimbleness, and in these times, this quality is essential to innovate.
Is all this too much to ask? Of late, too often, I read are we expecting too much from young entrepreneurs. Was too much expected of Alexander when he became king at 20 and set out to conquer the world? Or of Akbar who inherited the empire at the age of 14? Being a founder, building a high-velocity organisation is no doubt high-pressure, but isn’t it a choice? Isn’t competing in the Olympics different from playing cricket in your backyard?
Over the years, I have a more nuanced take about winning itself.
Winning doesn’t always mean being first; winning means you are doing better than you have done before.

 

Q.“We tend to put pressure on the young entrepreneurs”. This statement, in light of the passage, can be considered as: 

Detailed Solution for CAT Practice Test - 17 - Question 14

In the paragraph, “Is all this too much....cricket in your backyard?”, the author raises questions that appear to send across a message that the pressure experienced by the young entrepreneurs are brought on by themselves due to the choices they made in order to win.
Therefore, it can be said that it is not our expectations but their responsibilities that put pressure on them. Thus, the given statement can be said to be probably false as our understanding on the matter is inferential.
Hence, the correct answer is option 4.

CAT Practice Test - 17 - Question 15

The passage given below is followed by a question. Choose the most appropriate answer to each question.

Is anyone a born entrepreneur, a born leader or born to win? I believe everyone is born to win but you have to make winning a habit. Think of this: a quitter never wins, a winner never quits. I have been privileged to share the dream and journey of many entrepreneurs. A few traits are habits with all of them. A winner is a believer. Belief in oneself and one’s convictions — that inner voice of core strength to pursue dreams despite difficulties.

Along the way everyone they meet gets swept up in their dreams and aspirations. This belief is rooted in a bigger purpose than self- satisfaction. Winners are great listeners. They learn continuously; they take inputs; they don’t think they have all the answers; and they are confident to have the humility to hear other points of view.

Listening is about striving for comprehension and making oneself open to possibilities. Winners value self-development. They constantly challenge themselves and set aggressive goals and often surpass expectations. They make organisational learning a priority. Winners are unstoppable. They don’t count hurdles. They don’t brood about failures and risks. They are activators.

They have a clear compass on why they keep trying and what they set out to achieve. Hence they have the edge to get up, dust themselves off, and set out again and again.

Winners are decisive. They chart the course and make choices all the time while being transparent and suffused with a clarity of purpose. They are less prone to procrastination. They are resilient to change and very effective in communicating their decisions. Another word for this is nimbleness, and in these times, this quality is essential to innovate.
Is all this too much to ask? Of late, too often, I read are we expecting too much from young entrepreneurs. Was too much expected of Alexander when he became king at 20 and set out to conquer the world? Or of Akbar who inherited the empire at the age of 14? Being a founder, building a high-velocity organisation is no doubt high-pressure, but isn’t it a choice? Isn’t competing in the Olympics different from playing cricket in your backyard?
Over the years, I have a more nuanced take about winning itself.
Winning doesn’t always mean being first; winning means you are doing better than you have done before.

 

Q.Which of the following is a possible source of the passage?

Detailed Solution for CAT Practice Test - 17 - Question 15

The language used in the passage is simple and does not involve complex technical terms, abbreviations, or jargons. Thus, option 1 is eliminated.
The nature of the passage is clearly not autobiographical as the author is not speaking about himself/herself. Thus, option 2 is eliminated.
The passage is not dealing with a person’s life. Thus, option 4 is eliminated.
The topic of the passage is inspirational and possibly could aid in reader’s personal development. Thus, the passage could be an excerpt from a motivational book. Hence, the correct answer is option 3.

CAT Practice Test - 17 - Question 16

A passage is followed by questions pertaining to the passage. Read the passage and answer the questions. Choose the most appropriate answer.

Folklorists often interpret the fairy tale Cinderella as the competition between the stepmother and the stepdaughter for resources, which may include the need to provide a dowry. Gioachino Rossini's opera La Cenerentola makes this economic basis explicit: Don Magnifico wishes to make his own daughters' dowry larger, to attract a grander match, which is impossible if he must provide a third dowry. One common penalty for the kidnapping of an unmarried woman was that the abductor had to provide the woman's dowry. Until the late 20th century this was sometimes called wreath money, or the breach of promise. Providing dowries for poor women was regarded as a form of charity by wealthier parishioners. The custom of Christmas stockings springs from a legend of St. Nicholas, in which he threw gold in the stockings of three poor sisters, thus providing for their dowries. St. Elizabeth of Portugal and St. Martin de Porres were particularly noted for providing such dowries, and the Archconfraternity of the Annunciation, a Roman charity dedicated to providing dowries, received the entire estate of Pope Urban VII. As the French crown provided dowries for many of the women persuaded to travel to New France for marriages and settlement there, they were known as filles du roi (daughters of the king). In some parts of Europe, land dowries were common. In the County of Bentheim, for instance, parents who had no sons might give a land dowry to their new son-in- law. It was commonly given with the condition that he take the surname of his bride, in order to continue the family name. The Portuguese crown gave two cities as dowry to the British Crown in 1661 when King Charles II of England, Scotland and Ireland married Catherine of Braganza, a princess of Portugal. They were Mumbai (Bombay) in India and Tangier in Morocco.

 

Q.Which of the following can be stated about Don Magnifico from the passage?
A. He is a miserly man who keeps his money close to his heart.
B. He hates his daughters and simply wants to many them off.
C. He has a social status equivalent to a king that requires him to pay a high dowry for his daughters.

Detailed Solution for CAT Practice Test - 17 - Question 16

The passage states that “Gioachino Rossini’s opera La Cenerentola makes this economic basis explicit: Don Magnifico wishes to make his own daughters’ dowry larger, to attract a grander match, which is impossible if he must provide a third dowry.” Statement A is incorrect since a miser would not want to give his daughters a large dowry.
Statement B with ‘hates’ and statement C with ‘equivalent to a king’ cannot be inferred form the data given in the passage.
Hence, the correct answer is option 4.

CAT Practice Test - 17 - Question 17

A passage is followed by questions pertaining to the passage. Read the passage and answer the questions. Choose the most appropriate answer.

Folklorists often interpret the fairy tale Cinderella as the competition between the stepmother and the stepdaughter for resources, which may include the need to provide a dowry. Gioachino Rossini's opera La Cenerentola makes this economic basis explicit: Don Magnifico wishes to make his own daughters' dowry larger, to attract a grander match, which is impossible if he must provide a third dowry. One common penalty for the kidnapping of an unmarried woman was that the abductor had to provide the woman's dowry. Until the late 20th century this was sometimes called wreath money, or the breach of promise. Providing dowries for poor women was regarded as a form of charity by wealthier parishioners. The custom of Christmas stockings springs from a legend of St. Nicholas, in which he threw gold in the stockings of three poor sisters, thus providing for their dowries. St. Elizabeth of Portugal and St. Martin de Porres were particularly noted for providing such dowries, and the Archconfraternity of the Annunciation, a Roman charity dedicated to providing dowries, received the entire estate of Pope Urban VII. As the French crown provided dowries for many of the women persuaded to travel to New France for marriages and settlement there, they were known as filles du roi (daughters of the king). In some parts of Europe, land dowries were common. In the County of Bentheim, for instance, parents who had no sons might give a land dowry to their new son-in- law. It was commonly given with the condition that he take the surname of his bride, in order to continue the family name. The Portuguese crown gave two cities as dowry to the British Crown in 1661 when King Charles II of England, Scotland and Ireland married Catherine of Braganza, a princess of Portugal. They were Mumbai (Bombay) in India and Tangier in Morocco.

 

Q.Which of the following statements is not a fact?

Detailed Solution for CAT Practice Test - 17 - Question 17

The passage states that “The custom of Christmas stockings springs from a legend of St. Nicholas, in which he threw gold in the stockings of three poor sisters, thus providing for their dowries.” The key word here is “legend” - it means “a nonhistorical or unverifiable story handed down by tradition from earlier times and popularly accepted as historical.” Thus, a legend is something that is not a fact.
Options 1,2, and 3 are facts according to the passage and can be eliminated.
Hence, the correct answer is option 4.

CAT Practice Test - 17 - Question 18

A passage is followed by questions pertaining to the passage. Read the passage and answer the questions. Choose the most appropriate answer.

Folklorists often interpret the fairy tale Cinderella as the competition between the stepmother and the stepdaughter for resources, which may include the need to provide a dowry. Gioachino Rossini's opera La Cenerentola makes this economic basis explicit: Don Magnifico wishes to make his own daughters' dowry larger, to attract a grander match, which is impossible if he must provide a third dowry. One common penalty for the kidnapping of an unmarried woman was that the abductor had to provide the woman's dowry. Until the late 20th century this was sometimes called wreath money, or the breach of promise. Providing dowries for poor women was regarded as a form of charity by wealthier parishioners. The custom of Christmas stockings springs from a legend of St. Nicholas, in which he threw gold in the stockings of three poor sisters, thus providing for their dowries. St. Elizabeth of Portugal and St. Martin de Porres were particularly noted for providing such dowries, and the Archconfraternity of the Annunciation, a Roman charity dedicated to providing dowries, received the entire estate of Pope Urban VII. As the French crown provided dowries for many of the women persuaded to travel to New France for marriages and settlement there, they were known as filles du roi (daughters of the king). In some parts of Europe, land dowries were common. In the County of Bentheim, for instance, parents who had no sons might give a land dowry to their new son-in- law. It was commonly given with the condition that he take the surname of his bride, in order to continue the family name. The Portuguese crown gave two cities as dowry to the British Crown in 1661 when King Charles II of England, Scotland and Ireland married Catherine of Braganza, a princess of Portugal. They were Mumbai (Bombay) in India and Tangier in Morocco.

 

Q.Which of the following could be an apt title for the passage?

Detailed Solution for CAT Practice Test - 17 - Question 18

The passage traces the historical path of dowry and its many forms in Europe.
Option 2 is incorrect as the passage does not debate whether it is good or bad, but maintains a neutral stand.
Option 3 is incorrect as India is only mentioned in passing and is not the focus of the passage.
Option 4 is incorrect as it paints dowry in a negative light, while the passage maintains an opinion that is strictly “neutral.” Hence, the correct answer is option 1.

CAT Practice Test - 17 - Question 19

The passage given below is followed by a question. Choose the most appropriate answer to each question.

Conventional wisdom for the future of India is that we must grow like China or Japan. That we must build large companies creating thousands of jobs and exporting goods. I believe that domestic consumption, not exports, will drive India’s growth. The economy will be services-led and not manufacturing-led like China or Japan. Small businesses will lead rather than large corporations.

India is the largest young country in an ageing world and will continue to have a young population for the next 25 years, whereas China has started ageing. Indians will either migrate or do outsourcing work. Care providers around the world will come from India. There will be doubling of GDP growth in housing, education, health - all services. Services are labour intensive and their incremental return on capital is much faster than manufacturing and then there will be services like tourism that create jobs.

Many economists have suggested that India should copy China, but it cannot. When China started its development journey, it had no established competition. Global overcapacity challenges India. For example, China has steel capacity of 822 million tonnes and India has 86 million tonnes. Recently, the Indian government had to protect its steel industry by introducing minimum import pricing. Normal competition without tariffs will be difficult in many sectors in Indian economy.

India’s free market for labour combined with single market for services is the reason why services is the country’s biggest growth area. The only place where India can achieve economy of scale is in services. This is apparent in the dramatic growth of service tax.
The set of programming interfaces built on the trifecta of government- created people’s bank account of Jan Dhan, Aadhaar no., and mobile phones - in brief JAM - enables paperless, presence-less and cashless transactions.

Dramatic consequences will follow creating thousands of startups and billions of dollars of capitalisation. Four shifts will happen. First, banking at scale because everything a bank can do, individuals can do on a mobile phone. Second, investment at scale - people can buy a mutual fund on the phone with one click. Third, credit at scale where entrepreneurs can get a loan with just a click by aggregating their own data. And fourth, skilling at scale - as platforms happen, India will have thousands, millions of people gathering skills to operate in this new economy with great strides in reading and math literacy happening at scale.
World trade may be shrinking and barriers may be emerging among nations, all making movement of labour difficult. India with its vast unified market, youthful labour force and growing digital platform-backed services alone is poised to build a new power economy.

 

Q.Which of the following is a plausible reason for the substantial  growth in service tax in India?

Detailed Solution for CAT Practice Test - 17 - Question 19

Option 2 can be concluded from the sentence, “....services is the country’s biggest growth area. “.
Option 1 cannot be corroborated from the passage.
Option 3 incorrectly relates the rising service tax with growth in startups. So, eliminate options 3 and 4.
Hence, the correct answer is option 2.

CAT Practice Test - 17 - Question 20

The passage given below is followed by a question. Choose the most appropriate answer to each question.

Conventional wisdom for the future of India is that we must grow like China or Japan. That we must build large companies creating thousands of jobs and exporting goods. I believe that domestic consumption, not exports, will drive India’s growth. The economy will be services-led and not manufacturing-led like China or Japan. Small businesses will lead rather than large corporations.

India is the largest young country in an ageing world and will continue to have a young population for the next 25 years, whereas China has started ageing. Indians will either migrate or do outsourcing work. Care providers around the world will come from India. There will be doubling of GDP growth in housing, education, health - all services. Services are labour intensive and their incremental return on capital is much faster than manufacturing and then there will be services like tourism that create jobs.

Many economists have suggested that India should copy China, but it cannot. When China started its development journey, it had no established competition. Global overcapacity challenges India. For example, China has steel capacity of 822 million tonnes and India has 86 million tonnes. Recently, the Indian government had to protect its steel industry by introducing minimum import pricing. Normal competition without tariffs will be difficult in many sectors in Indian economy.

India’s free market for labour combined with single market for services is the reason why services is the country’s biggest growth area. The only place where India can achieve economy of scale is in services. This is apparent in the dramatic growth of service tax.
The set of programming interfaces built on the trifecta of government- created people’s bank account of Jan Dhan, Aadhaar no., and mobile phones - in brief JAM - enables paperless, presence-less and cashless transactions.

Dramatic consequences will follow creating thousands of startups and billions of dollars of capitalisation. Four shifts will happen. First, banking at scale because everything a bank can do, individuals can do on a mobile phone. Second, investment at scale - people can buy a mutual fund on the phone with one click. Third, credit at scale where entrepreneurs can get a loan with just a click by aggregating their own data. And fourth, skilling at scale - as platforms happen, India will have thousands, millions of people gathering skills to operate in this new economy with great strides in reading and math literacy happening at scale.
World trade may be shrinking and barriers may be emerging among nations, all making movement of labour difficult. India with its vast unified market, youthful labour force and growing digital platform-backed services alone is poised to build a new power economy.

 

Q.Which of the following best describes author’s style of writing?

Detailed Solution for CAT Practice Test - 17 - Question 20

Specific data in terms of numbers/percentages/fractions or the like are not excessively used in the passage and they certainly do not form the backbone of the passage and hence the passage cannot be called data driven. Option 1 is eliminated.
The author begins by taking a stand on the topic and then explaining why his view is stronger as compared to the conventional one. This shows that the author is being argumentative and not analytical only. So, eliminate option 3.
The author is not just giving factual information regarding service sector here but comparing it with other sectors, reasoning, and trying to validate his argument. Thus, author’s style cannot be completely described by the word “descriptive”. Thus, option 4 is eliminated.
Hence, the correct answer is option 2.

CAT Practice Test - 17 - Question 21

The passage given below is followed by a question. Choose the most appropriate answer to each question.

Conventional wisdom for the future of India is that we must grow like China or Japan. That we must build large companies creating thousands of jobs and exporting goods. I believe that domestic consumption, not exports, will drive India’s growth. The economy will be services-led and not manufacturing-led like China or Japan. Small businesses will lead rather than large corporations.

India is the largest young country in an ageing world and will continue to have a young population for the next 25 years, whereas China has started ageing. Indians will either migrate or do outsourcing work. Care providers around the world will come from India. There will be doubling of GDP growth in housing, education, health - all services. Services are labour intensive and their incremental return on capital is much faster than manufacturing and then there will be services like tourism that create jobs.

Many economists have suggested that India should copy China, but it cannot. When China started its development journey, it had no established competition. Global overcapacity challenges India. For example, China has steel capacity of 822 million tonnes and India has 86 million tonnes. Recently, the Indian government had to protect its steel industry by introducing minimum import pricing. Normal competition without tariffs will be difficult in many sectors in Indian economy.

India’s free market for labour combined with single market for services is the reason why services is the country’s biggest growth area. The only place where India can achieve economy of scale is in services. This is apparent in the dramatic growth of service tax.
The set of programming interfaces built on the trifecta of government- created people’s bank account of Jan Dhan, Aadhaar no., and mobile phones - in brief JAM - enables paperless, presence-less and cashless transactions.

Dramatic consequences will follow creating thousands of startups and billions of dollars of capitalisation. Four shifts will happen. First, banking at scale because everything a bank can do, individuals can do on a mobile phone. Second, investment at scale - people can buy a mutual fund on the phone with one click. Third, credit at scale where entrepreneurs can get a loan with just a click by aggregating their own data. And fourth, skilling at scale - as platforms happen, India will have thousands, millions of people gathering skills to operate in this new economy with great strides in reading and math literacy happening at scale.
World trade may be shrinking and barriers may be emerging among nations, all making movement of labour difficult. India with its vast unified market, youthful labour force and growing digital platform-backed services alone is poised to build a new power economy.

 

Q.From which source the passage has been taken possibly? 

Detailed Solution for CAT Practice Test - 17 - Question 21

The language used in the passage is simple and does not involve too many complex technical terms, abbreviations, or jargons. Also, the author does not seem to be diving too deep into the subject of discussion. Thus, option 1 is eliminated.
The topic of the passage is not inspirational and could not possibly aid in reader’s personal development. Thus, option 2 is eliminated.
The passage does not present any statistical data and thus, cannot be a part of a financial report.
Hence, the correct answer is option 3.

CAT Practice Test - 17 - Question 22

The passage given below is followed by a question. Choose the most appropriate answer to each question.

Conventional wisdom for the future of India is that we must grow like China or Japan. That we must build large companies creating thousands of jobs and exporting goods. I believe that domestic consumption, not exports, will drive India’s growth. The economy will be services-led and not manufacturing-led like China or Japan. Small businesses will lead rather than large corporations.

India is the largest young country in an ageing world and will continue to have a young population for the next 25 years, whereas China has started ageing. Indians will either migrate or do outsourcing work. Care providers around the world will come from India. There will be doubling of GDP growth in housing, education, health - all services. Services are labour intensive and their incremental return on capital is much faster than manufacturing and then there will be services like tourism that create jobs.

Many economists have suggested that India should copy China, but it cannot. When China started its development journey, it had no established competition. Global overcapacity challenges India. For example, China has steel capacity of 822 million tonnes and India has 86 million tonnes. Recently, the Indian government had to protect its steel industry by introducing minimum import pricing. Normal competition without tariffs will be difficult in many sectors in Indian economy.

India’s free market for labour combined with single market for services is the reason why services is the country’s biggest growth area. The only place where India can achieve economy of scale is in services. This is apparent in the dramatic growth of service tax.
The set of programming interfaces built on the trifecta of government- created people’s bank account of Jan Dhan, Aadhaar no., and mobile phones - in brief JAM - enables paperless, presence-less and cashless transactions.

Dramatic consequences will follow creating thousands of startups and billions of dollars of capitalisation. Four shifts will happen. First, banking at scale because everything a bank can do, individuals can do on a mobile phone. Second, investment at scale - people can buy a mutual fund on the phone with one click. Third, credit at scale where entrepreneurs can get a loan with just a click by aggregating their own data. And fourth, skilling at scale - as platforms happen, India will have thousands, millions of people gathering skills to operate in this new economy with great strides in reading and math literacy happening at scale.
World trade may be shrinking and barriers may be emerging among nations, all making movement of labour difficult. India with its vast unified market, youthful labour force and growing digital platform-backed services alone is poised to build a new power economy.

 

Q.Author’s point of view in the above passage can be called as which of the following?

Detailed Solution for CAT Practice Test - 17 - Question 22

There is no undue use of exaggeration, exhibitionism, or bombasting in the passage, and hence author’s view can’t be termed as rhetorical. Thus, option 1 is eliminated.
The author’s argument is not in line with the the set form or convention regarding India’s future and hence it can’t be called stereotypical. Thus, option 2 is eliminated.
Though the author takes a stand in the very beginning, he discusses the merit of his argument by comparing it with the opposing view (e.g. example of China) and is open to go into the merit of the opposing argument also and seems rather flexible. Thus, his view can’t be termed as rigid. Hence, option 3 is eliminated.
The first paragraph in the passage itself clearly suggests that author’s view is challenging the conventional wisdom. Thus, author’s view is unconventional.
Hence, the correct answer is option 4.

CAT Practice Test - 17 - Question 23

The passage given below is followed by a question. Choose the most appropriate answer to each question.

Conventional wisdom for the future of India is that we must grow like China or Japan. That we must build large companies creating thousands of jobs and exporting goods. I believe that domestic consumption, not exports, will drive India’s growth. The economy will be services-led and not manufacturing-led like China or Japan. Small businesses will lead rather than large corporations.

India is the largest young country in an ageing world and will continue to have a young population for the next 25 years, whereas China has started ageing. Indians will either migrate or do outsourcing work. Care providers around the world will come from India. There will be doubling of GDP growth in housing, education, health - all services. Services are labour intensive and their incremental return on capital is much faster than manufacturing and then there will be services like tourism that create jobs.

Many economists have suggested that India should copy China, but it cannot. When China started its development journey, it had no established competition. Global overcapacity challenges India. For example, China has steel capacity of 822 million tonnes and India has 86 million tonnes. Recently, the Indian government had to protect its steel industry by introducing minimum import pricing. Normal competition without tariffs will be difficult in many sectors in Indian economy.

India’s free market for labour combined with single market for services is the reason why services is the country’s biggest growth area. The only place where India can achieve economy of scale is in services. This is apparent in the dramatic growth of service tax.
The set of programming interfaces built on the trifecta of government- created people’s bank account of Jan Dhan, Aadhaar no., and mobile phones - in brief JAM - enables paperless, presence-less and cashless transactions.

Dramatic consequences will follow creating thousands of startups and billions of dollars of capitalisation. Four shifts will happen. First, banking at scale because everything a bank can do, individuals can do on a mobile phone. Second, investment at scale - people can buy a mutual fund on the phone with one click. Third, credit at scale where entrepreneurs can get a loan with just a click by aggregating their own data. And fourth, skilling at scale - as platforms happen, India will have thousands, millions of people gathering skills to operate in this new economy with great strides in reading and math literacy happening at scale.
World trade may be shrinking and barriers may be emerging among nations, all making movement of labour difficult. India with its vast unified market, youthful labour force and growing digital platform-backed services alone is poised to build a new power economy.

 

Q.Which of the following statements, if true, strengthens the argument in the above passage?
A. According to a survey, the job applications for bank jobs are on the rise as compared to the engineering sector.
B. China's service sector growth has stagnated.

Detailed Solution for CAT Practice Test - 17 - Question 23

Statement A indicates that Indian youth is possibly anticipating growth in service sector, which corroborates the Indian service sector growth story projected by the author. Thus, statement A is a strengthening statement.
Author’s argument is about India’s unique position in service sector. The passage only states that China is a manufacturing- led country, so its position in the service sector cannot be inferred from the passage. Thus, China’s service sector stagnation does not affect author’s argument in any way and can be called a generic statement.
Thus, only statement A is strengthening.
Hence, the correct answer is option 1.

CAT Practice Test - 17 - Question 24

The passage given below is followed by a question. Choose the most appropriate answer to each question.

Conventional wisdom for the future of India is that we must grow like China or Japan. That we must build large companies creating thousands of jobs and exporting goods. I believe that domestic consumption, not exports, will drive India’s growth. The economy will be services-led and not manufacturing-led like China or Japan. Small businesses will lead rather than large corporations.

India is the largest young country in an ageing world and will continue to have a young population for the next 25 years, whereas China has started ageing. Indians will either migrate or do outsourcing work. Care providers around the world will come from India. There will be doubling of GDP growth in housing, education, health - all services. Services are labour intensive and their incremental return on capital is much faster than manufacturing and then there will be services like tourism that create jobs.

Many economists have suggested that India should copy China, but it cannot. When China started its development journey, it had no established competition. Global overcapacity challenges India. For example, China has steel capacity of 822 million tonnes and India has 86 million tonnes. Recently, the Indian government had to protect its steel industry by introducing minimum import pricing. Normal competition without tariffs will be difficult in many sectors in Indian economy.

India’s free market for labour combined with single market for services is the reason why services is the country’s biggest growth area. The only place where India can achieve economy of scale is in services. This is apparent in the dramatic growth of service tax.
The set of programming interfaces built on the trifecta of government- created people’s bank account of Jan Dhan, Aadhaar no., and mobile phones - in brief JAM - enables paperless, presence-less and cashless transactions.

Dramatic consequences will follow creating thousands of startups and billions of dollars of capitalisation. Four shifts will happen. First, banking at scale because everything a bank can do, individuals can do on a mobile phone. Second, investment at scale - people can buy a mutual fund on the phone with one click. Third, credit at scale where entrepreneurs can get a loan with just a click by aggregating their own data. And fourth, skilling at scale - as platforms happen, India will have thousands, millions of people gathering skills to operate in this new economy with great strides in reading and math literacy happening at scale.
World trade may be shrinking and barriers may be emerging among nations, all making movement of labour difficult. India with its vast unified market, youthful labour force and growing digital platform-backed services alone is poised to build a new power economy.

 

Q.Consider the following statement: “India is poised to be a big player in the service sector.” Which of the following statements, if true, would weaken this statement?
A. Due to various factors, China’s steel production is likely to increase significantly while that of India is going to increase only marginally.
B. The capital flow or investment pattern is unfavourable for the service sector.

Detailed Solution for CAT Practice Test - 17 - Question 24

Production of steel obviously relates to manufacturing sector and this is rather irrelevant with respect to India’s service sector growth. Thus, statement A is neither a strengthening nor a weakening statement and hence is eliminated.
Statement B indicates that there has been scanty capital investment in the service sector which is a hurdle in the growth of the sector. Thus, statement B is weakening statement.
Hence, the correct answer is option 2.

*Answer can only contain numeric values
CAT Practice Test - 17 - Question 25

Q.The question below consists of a paragraph in which the first and the last sentences are fixed. Rearrange the intermediate sentences to form a coherent paragraph and enter the correct sequence in the box provided.
1. From my reading I know that most civilized successors of early cultures smile at the primitive ignorance of their ancestors.

2. Astonishingly, the land of my ancestors is little changed from that of a thousand years ago.

3. Yes, the modem buildings have sprung up but there is still little consideration for women.

4. Human society eagerly rushes to embrace the knowledge and change.

5. As civilization advances, the fear of freedom for the individual is overcome through enlightenment.

6. Consideration for the quality of lives of women still receives a shrug of indifference.


Detailed Solution for CAT Practice Test - 17 - Question 25

Statement 1 talks about the civilized successors of early cultures and statement 5 begins with “As civilization advances” which forms a link with 5. Thus, 1-5 make a pair. “Enlightenment” in statement 5 connects with “human society eagerly rushes to embrace the knowledge” in statement 4. Statement 2 gives a grim picture of the presently changed land pattern and statement 3 says what exactly has changed. Thus, 2-3 form a pair.
Hence, the correct sequence is 5423.

*Answer can only contain numeric values
CAT Practice Test - 17 - Question 26

The question below consists of a paragraph in which the first and the last sentences are fixed. Rearrange the intermediate sentences to form a coherent paragraph and enter the correct sequence in the box provided.
1. The word secular was inserted into the Preamble during emergency which implies equality of all religions and religious tolerance.

2. Every person has the right to preach, practice and propagate any religion they choose.

3. India, therefore does not have an official state religion.

4. No religious instruction is imparted in government or government-aided schools.

5. All citizens, irrespective of their religious beliefs are equal in the eyes of law.

6. Nevertheless, general information about all established world religions is imparted as part of the course in Sociology, without giving any importance to any one religion or the others.


Detailed Solution for CAT Practice Test - 17 - Question 26

The opening statement talks about the incorporation of “secular” into the Preamble and religious equality. This can be linked to statement 3 which justifies why India being a secular state, has no official religion. Statement 2 follows, as it goes on to define what “secular” really means. Statement 4 links well with the last statement, where religion in schools is the common theme; statement 4 mentions that no religion is forced in schools and the last statement mentions the availability of general information related to world religions in schools with no bias. Therefore, by elimination, only statement 5 can follow after statement 2.
Hence, the correct sequence is 3254.

*Answer can only contain numeric values
CAT Practice Test - 17 - Question 27

The question below consists of a set of labelled sentences. These sentences, when properly sequenced, form a coherent paragraph. Choose the most logical order of sentences from the options.

1. Politicians, so inured to the trivial crises of normal politics, often lack the ability to recognize when there is real danger.

2. Why does democracy everywhere seem to be in crisis, unable to deliver what its citizens want?

3. Every time democracy enters a new crisis, its friends despair.

4. David Runciman argues this is because democracy invariably disappoints.

5. It struggles to meet the challenge and grasp control of events.


Detailed Solution for CAT Practice Test - 17 - Question 27

The paragraph pertains to the reasons why democracy fails. Statement 2 makes for a more appropriate introductory statement in comparison to statements 1 and 3 since it puts forth the question which the other statements try to answer.
Statement 4 ought to follow statement 2 since it puts forth David Runciman's judgment on the question put forth by statement 2. Statement 3 will follow statement 4 since it puts forth a reason why “... democracy invariably disappoints ...”.
Statement 5 further elaborates on the “crisis” situation referred to in statement 3.
Statement 1 makes for an appropriate conclusion since it summarizes the reaction of the political leadership to the “crisis” mentioned.
Hence, the correct sequence is 24351.

*Answer can only contain numeric values
CAT Practice Test - 17 - Question 28

Carefully read the statements in the questions below and arrange them in a logical order.

1. The man was suddenly taken by a violent spasm of coughing.

2. The two adversaries faced each other with clenched fists and fuming eyes.

3. It had been 7 years into their marriage and it had degenerated beyond a point.

4. The fight broke out over his refusal to help in the household chores.

5. One could notice traces of concern in her eyes, all hope was not lost then.


Detailed Solution for CAT Practice Test - 17 - Question 28

Statement 3 is the introductory statement as it introduces the cause for the fight- the marriage had degenerated. 4-2 are a pair since 4 introduces the “fight” and 2 with “clenched fists and fuming eyes” indicate the aggression between the two. 1-5 are a pair since statement 1 mentions the man to be ill- “violent spasm of coughing” which links well with statement 5 which mention the “concern” in his wife's eyes.
Hence, the correct sequence is 34215.

*Answer can only contain numeric values
CAT Practice Test - 17 - Question 29

Five sentences are given below labeled (1), (2), (3), (4) and (5). Of these, four sentences need to be arranged in a logical order to form a coherent paragraph/passage. Pick out the sentence that does not fit the sequence.
1. What must we know in economics to say that we know economics is a substantially political and not a purely epistemic issue.

2. Even more political than the content and processes of education are the aims of education.

3. Education that aims to develop autonomous, critical thinking individuals and to help develop a just and democratic society is sharply political.

4. In fact, the aims of education shape the processes and content of education, including significantly determining their political tilt.

5. And as sharply political would be education that aims to develop individuals who are not questioning but conforming to some existing order.


Detailed Solution for CAT Practice Test - 17 - Question 29

Statement 2 introduces the topic of the paragraph - aims of education being political in nature. Statements 3 with “Education that aims” relates to “aims of education” in statement 2. Hence, 3 follows 2. Similarly, “sharply political” in statement 3 connects with “as sharply political” in statement 5. Statement 4 also belongs to the paragraph as it talks about the “aims of education”- the topic of the paragraph.
Statement 1 which talks about economics as a political subject in specific does not quite fit in with the rest of the statements which talk about the influence of politics on education in general.
Hence, the correct answer is 1.

*Answer can only contain numeric values
CAT Practice Test - 17 - Question 30

Five sentences are given below labeled (1), (2), (3), (4) and (5). Of these, four sentences need to be arranged in a logical order to form a coherent paragraph/passage. Pick out the sentence that does not fit the sequence.

1. Many are hesitant that they won't be able to fully adjust, and so wonder if it might be possible to stay with an Amish family for a week or two, just to try out the lifestyle.

2. The Amish don't have any spiritual attachment to a geographical location, the way Jews have to Jerusalem or Mormons to Salt Lake City; this spot, along with Lancaster County, Pennsylvania, is probably the closest they come to an idea of God’s Country.

3. Many say they’ve wanted to become Amish for “as long as [they] could remember,” though most of them say they have only seen Amish people on a few occasions, and don’t know much, if anything at all, about Amish theology.

4. Some talk about wanting to find an Amish partner, others, about the fear they won’t be accepted into the community because they are single parents, or divorced, or have tattoos or once dabbled in drugs.

5. Although a few commenters say they’ve taken the initiative to make their own lives more Plain- given up television, say, or started to dress more modestly-most of them appear to be banking on integration into the community to transform them, like alcoholics who decide to wait until detox before examining the deeper motivations behind their drinking.


Detailed Solution for CAT Practice Test - 17 - Question 30

Statement 3 introduces the topic of the paragraph - people who want to embrace the Amish lifestyle. Statements 4, 1, and 5 in that order further discuss the reasons or ways in which people are trying/thinking about to explore and experience the Amish lifestyle, or why they can’t.
Only statement 2, which talks about the Amish people themselves, does not fit in with the rest of the statements which are about people who are desirous of joining/experiencing the community.
Hence, the correct answer is 2.

*Answer can only contain numeric values
CAT Practice Test - 17 - Question 31

Five sentences are given below labeled (1), (2), (3), (4) and (5). Of these, four sentences need to be arranged in a logical order to form a coherent paragraph/passage. Pick out the sentence that does not fit the sequence.

1. The sharp drop in inflation, attributable largely to the collapse in world crude oil prices, opened a window of opportunity that is a real blessing.

2. It created the space for a counter-cyclical budget that could have used fiscal expansion as a stimulus for growth.

3. However, revenue receipts from taxes were squeezed, following acceptance of the Finance Commission recommendations, as the centre’s share of tax revenues dropped from 68% in 2015-16 to 58% in 2016-17.

4. The decision to peg the fiscal deficit at 3.5% of GDP became the keystone in the arch for the budget exercise, for it imposed a ceiling on the amount by which the government could allow its expenditure to exceed receipts.

5. The allocations for expenditure on salaries and pensions were bound to jump significantly, on account of the Pay Commission recommendations and the one-rank-one-pension decision for the armed forces.


Detailed Solution for CAT Practice Test - 17 - Question 31

Statement 1 is the only standalone sentence in the set, hence it must start the paragraph. The “space for a counter-cyclical budget” in statement 2 refers to the “window of opportunity” in statement 1. Hence 1-2 form a link. In statement 3, “however” presents a contradictory view to this optimistic aspect of “drop in inflation”. Therefore, 3 ought to follow 2. Statement 5 continues with this negative tone. Only statement 4 which gives a feature of the budget exercise is out of context and does not fit in the logical flow.
Hence, the correct answer is 4.

*Answer can only contain numeric values
CAT Practice Test - 17 - Question 32

Four sentences are given below labeled (1), (2), (3) and (4). Of these, three sentences need to be arranged in a logical order to form a coherent paragraph/passage. Pick out the sentence that does not fit the sequence.

1. The European Union and the United States, both of whom could have pushed for our inclusion, have their own incentives to placate Turkey, including its cooperation with Europe’s response to the refugee crisis and a need for Turkish support in the military campaign against the Islamic State.

2. We understand the demands of realpolitik, but the exclusion of Rojava from the U.N. talks is shortsighted and unjust.

3. Turkey has tried to legitimate its opposition with propaganda falsely depicting Rojava as an ethnic project for Kurdish dominance that aims to divide Syria.

4. This system could be a model for all of Syria, a country where any functioning democratic system would have to include all ethnic groups and religions in order to survive.


Detailed Solution for CAT Practice Test - 17 - Question 32

Statements 1 and 2 form a link - statement 2 tells us whose inclusion the EU and the US should have been pushed for. Statement 3 follows by telling us who is opposing the inclusion of Rojava in the UN talks and how. Thus, a sequence is formed - 1-2-3. Statement 4 which talks about a model system for inclusion of ethnic groups in Syria to make it more democratic, does not connect with the rest of the statements.
Hence, the correct answer is 4.

*Answer can only contain numeric values
CAT Practice Test - 17 - Question 33

Five sentences are given below labeled (1), (2), (3), (4) and (5). Of these, four sentences need to be arranged in a logical order to form a coherent paragraph/passage. Pick out the sentence that does not fit the sequence.

1. The idea of self-development is not new but a legacy of Confucian culture, where self-development is required to foster the development of others, and therefore create an overall more “harmonious” society.

2. China’s income distribution is, in fact, among the worst in the world; China's Gini coefficient for income, which is used to measure inequality, was 0.49 in 2012.

3. What is new, however, is that in today’s context, the wish for self-development is driven by the need to succeed.

4. The self-help industry, with its Chicken Soup for the Soul books and motivational classes, offers help to those who are in a fervent pursuit of happiness and well-being.

5. As E. Hendriks puts it, “Chinese urbanites are driven by the fear of losing out,” which is also why driving an Uber car in order to “go out and learn” is not so different from going out and attending a class on nutrition.


Detailed Solution for CAT Practice Test - 17 - Question 33

Statement 1 introduces the topic of the paragraph - self-development and says that the topic is not new but is a legacy of some culture. Statement 3 then continues by discussing what actually is new, which is self-development being driven by the desire to succeed.
Statements 5 with the phrase “fear of losing out” connects with "the need to succeed" in statement 3. Statement 4 says how the help is provided to those who are seeking success.
Statement 2, which speaks about China’s income distribution, does not fit in logically with the other statements.
Hence, the correct answer is 2.

*Answer can only contain numeric values
CAT Practice Test - 17 - Question 34

The question below contains a paragraph with a missing sentence or part of a sentence. Choose the option that most logically completes the paragraph.

The subtitles that appear under most foreign-language films in British and American movie theaters tend not to call attention to themselves. Moviegoers only notice them when something goes wrong— when they’ve been rendered extremely large, or when they’ve been colored a sickly yellow, or when they disappear against a white object, fall out of sync with the dialogue, flit abruptly to another region of the screen or bear the goofily garbled translations of which a Google search for “bad subtitles” gives you hundreds of examples.

1. As a visual element, the subtitle Is extraneous, replaceable and unassimilated into everything else in a given frame.

2. Thus, the visual rendition of subtitles is significant in terms of the words perceived by the reader.

3. Possibly because they’re designed to go unremarked, subtitle translations remain one of the least studied and most overlooked features a film can have.

4. One can destroy a film in several ways: cut it up, bum it—or subtitle it.


Detailed Solution for CAT Practice Test - 17 - Question 34

The passage talks about a unique attribute of movies called subtitles and how movie watchers tend to treat the subtitles when they are being displayed on the screen.
Option 1 describes a subtitle and does not exactly qualify as a suitable conclusion.
Option 2 is not a suitable conclusion as the colour aspect of the subtitle is a point brought up in the passage and is not the main topic. Option 3 properly sums up the points mentioned in the passage by highlighting the position of the subtitles in the movies.
Option 4 is absurd and shows “subtitles” in an extreme negative light. Hence, the correct answer is option 3.

*Answer can only contain numeric values
CAT Practice Test - 17 - Question 35

Complete the paragraph with the appropriate option.

Just because you don’t work in an office or a factory doesn’t automatically mean you don’t work. Says the Office for National Statistics. For the first time the ONS has officially quantified the home production economy. If government officers spent more time reading history, they would possibly have worked this out a long time ago.
The word “economy” would have tipped them off. As would the term “housework”. Economy is derived from Greek, meaning management of a household or family. And in English, until the Industrial Revolution, housework meant the work of the house -

Which option best completes the above paragraph?

1. wherein family members give up their dreams to sustain their family.

2. to look after the kids and maintain a social image.

3. where all family members contribute equally into the household chores.

4. the work that needed to be done to keep a family afloat.


Detailed Solution for CAT Practice Test - 17 - Question 35

The passage talks about the significance of housework. Only option 4 elaborately defines housework where work is aptly related to house and family, and completes the paragraph.
Option 1 is very extreme with respect to the subject of the passage and requires additional explanation to justify its claim.
Option 2 is very specific and brings “social image” into the picture which is uncalled for.
Option 3 does not define the aspect of household work as hinted at by the passage.
Hence, the correct answer is option 4.

CAT Practice Test - 17 - Question 36

The passage given below is followed by a question. Choose the most appropriate answer to each question.

Alzheimer's disease is the most common form of dementia. This incurable, degenerative, and terminal disease was first described by German psychiatrist and neuropathologist Alois Alzheimer in 1906 and was named after him. Generally it is diagnosed in people over 65 years of age, although the less-prevalent early-onset Alzheimer's can occur much earlier. The first symptoms are often mistaken as related to ageing or stress. Detailed neuropsychological testing can reveal mild cognitive difficulties up to eight years before a person fulfills the clinical criteria for diagnosis of AD. These early symptoms can affect the most complex daily living activities. The most noticeable deficit is memory loss, which shows up as difficulty in remembering recently learned facts and inability to acquire new information. Subtle problems with the executive functions of attentiveness, planning, flexibility, and abstract thinking, or impairments in semantic memory can also be symptomatic of the early stages of AD. Apathy can be observed at this stage, and remains the most persistent neuropsychiatric symptom throughout the course of the disease. The preclinical stage of the disease has also been termed mild cognitive impairment, but there is still debate on whether this term corresponds to a different diagnostic entity by itself or just a first step of the disease.


In people with AD the increasing impairment of learning and memory eventually leads to a definitive diagnosis. In a small proportion of them, difficulties with language, executive functions, perception (agnosia), or execution of movements (apraxia) are more prominent than memory problems. AD does not affect all memory capacities equally. Older memories of the person's life (episodic memory), facts learned (semantic memory), and implicit memory are affected to a lesser degree than new facts or memories. Language problems are mainly characterised by a shrinking vocabulary and decreased word fluency, which lead to a general impoverishment of oral and written language.

Progressive deterioration eventually hinders independence. Speech difficulties become evident due to an inability to recall vocabulary, which leads to frequent incorrect word substitutions. Reading and writing skills are also progressively lost. Complex motor sequences become less coordinated as time passes, reducing the ability to perform most normal daily living activities. During this phase, memory problems worsen, and the person may fail to recognise close relatives. Long-term memory, which was previously intact, becomes impaired and behavioural changes become more prevalent. Common neuropsychiatric manifestations are wandering, sundowning, irritability and labile affect, leading to crying, outbursts of unpremeditated aggression, or resistance to caregiving. During this last stage of AD, the patient is completely dependent upon caregivers. Language is reduced to simple phrases or even single words, eventually leading to complete loss of speech. Despite the loss of verbal language abilities, patients can often understand and return emotional signals. Finally comes death, usually caused directly by some external factor such as pressure ulcers or pneumonia, not by the disease itself.

 

Q.From the information mentioned in the passage, about “apathy”, it can be concluded that the patient:

Detailed Solution for CAT Practice Test - 17 - Question 36

“Apathy” means ‘lack of interest in or concern for things that others find moving or exciting’.
The option which is closest to this meaning is option 2. Option 1 is too strong.
Options 3 is virtually opposite to the meaning of “apathy”. “Claustrophobia” is ‘an abnormal fear of being in enclosed or narrow places’. It is not applicable in this case. Hence, the correct answer is option 2.

CAT Practice Test - 17 - Question 37

The passage given below is followed by a question. Choose the most appropriate answer to each question.

Alzheimer's disease is the most common form of dementia. This incurable, degenerative, and terminal disease was first described by German psychiatrist and neuropathologist Alois Alzheimer in 1906 and was named after him. Generally it is diagnosed in people over 65 years of age, although the less-prevalent early-onset Alzheimer's can occur much earlier. The first symptoms are often mistaken as related to ageing or stress. Detailed neuropsychological testing can reveal mild cognitive difficulties up to eight years before a person fulfills the clinical criteria for diagnosis of AD. These early symptoms can affect the most complex daily living activities. The most noticeable deficit is memory loss, which shows up as difficulty in remembering recently learned facts and inability to acquire new information. Subtle problems with the executive functions of attentiveness, planning, flexibility, and abstract thinking, or impairments in semantic memory can also be symptomatic of the early stages of AD. Apathy can be observed at this stage, and remains the most persistent neuropsychiatric symptom throughout the course of the disease. The preclinical stage of the disease has also been termed mild cognitive impairment, but there is still debate on whether this term corresponds to a different diagnostic entity by itself or just a first step of the disease.


In people with AD the increasing impairment of learning and memory eventually leads to a definitive diagnosis. In a small proportion of them, difficulties with language, executive functions, perception (agnosia), or execution of movements (apraxia) are more prominent than memory problems. AD does not affect all memory capacities equally. Older memories of the person's life (episodic memory), facts learned (semantic memory), and implicit memory are affected to a lesser degree than new facts or memories. Language problems are mainly characterised by a shrinking vocabulary and decreased word fluency, which lead to a general impoverishment of oral and written language.

Progressive deterioration eventually hinders independence. Speech difficulties become evident due to an inability to recall vocabulary, which leads to frequent incorrect word substitutions. Reading and writing skills are also progressively lost. Complex motor sequences become less coordinated as time passes, reducing the ability to perform most normal daily living activities. During this phase, memory problems worsen, and the person may fail to recognise close relatives. Long-term memory, which was previously intact, becomes impaired and behavioural changes become more prevalent. Common neuropsychiatric manifestations are wandering, sundowning, irritability and labile affect, leading to crying, outbursts of unpremeditated aggression, or resistance to caregiving. During this last stage of AD, the patient is completely dependent upon caregivers. Language is reduced to simple phrases or even single words, eventually leading to complete loss of speech. Despite the loss of verbal language abilities, patients can often understand and return emotional signals. Finally comes death, usually caused directly by some external factor such as pressure ulcers or pneumonia, not by the disease itself.

 

Q.According to the passage, which of the following is not true? 

Detailed Solution for CAT Practice Test - 17 - Question 37

Option 1 is not true since speech difficulties are not among the first symptoms. Moreover, the passage does not comment on symptoms due to ageing or stress.
Option 2 is true since the passage mentions, “Complex motor sequences become less coordinated as time passes, reducing the ability to perform most normal daily living activities.” Option 4 is true as can be deciphered from the following, “Language problems are mainly characterised by a shrinking vocabulary and decreased word fluency.” Option 3 is true as can be deciphered from the following extract from the passage, “Subtle problems with the executive functions of attentiveness, planning, flexibility, and abstract thinking, or impairments in semantic memory can also be symptomatic of the early stages of AD.” Hence, the correct answer is option 1.

CAT Practice Test - 17 - Question 38

The passage given below is followed by a question. Choose the most appropriate answer to each question.

Alzheimer's disease is the most common form of dementia. This incurable, degenerative, and terminal disease was first described by German psychiatrist and neuropathologist Alois Alzheimer in 1906 and was named after him. Generally it is diagnosed in people over 65 years of age, although the less-prevalent early-onset Alzheimer's can occur much earlier. The first symptoms are often mistaken as related to ageing or stress. Detailed neuropsychological testing can reveal mild cognitive difficulties up to eight years before a person fulfills the clinical criteria for diagnosis of AD. These early symptoms can affect the most complex daily living activities. The most noticeable deficit is memory loss, which shows up as difficulty in remembering recently learned facts and inability to acquire new information. Subtle problems with the executive functions of attentiveness, planning, flexibility, and abstract thinking, or impairments in semantic memory can also be symptomatic of the early stages of AD. Apathy can be observed at this stage, and remains the most persistent neuropsychiatric symptom throughout the course of the disease. The preclinical stage of the disease has also been termed mild cognitive impairment, but there is still debate on whether this term corresponds to a different diagnostic entity by itself or just a first step of the disease.


In people with AD the increasing impairment of learning and memory eventually leads to a definitive diagnosis. In a small proportion of them, difficulties with language, executive functions, perception (agnosia), or execution of movements (apraxia) are more prominent than memory problems. AD does not affect all memory capacities equally. Older memories of the person's life (episodic memory), facts learned (semantic memory), and implicit memory are affected to a lesser degree than new facts or memories. Language problems are mainly characterised by a shrinking vocabulary and decreased word fluency, which lead to a general impoverishment of oral and written language.

Progressive deterioration eventually hinders independence. Speech difficulties become evident due to an inability to recall vocabulary, which leads to frequent incorrect word substitutions. Reading and writing skills are also progressively lost. Complex motor sequences become less coordinated as time passes, reducing the ability to perform most normal daily living activities. During this phase, memory problems worsen, and the person may fail to recognise close relatives. Long-term memory, which was previously intact, becomes impaired and behavioural changes become more prevalent. Common neuropsychiatric manifestations are wandering, sundowning, irritability and labile affect, leading to crying, outbursts of unpremeditated aggression, or resistance to caregiving. During this last stage of AD, the patient is completely dependent upon caregivers. Language is reduced to simple phrases or even single words, eventually leading to complete loss of speech. Despite the loss of verbal language abilities, patients can often understand and return emotional signals. Finally comes death, usually caused directly by some external factor such as pressure ulcers or pneumonia, not by the disease itself.

 

Q.Which of the following is not stated in the passage?

Detailed Solution for CAT Practice Test - 17 - Question 38

Options 1,2 and 3 are statements directly picked up from the passage.
The last paragraph of the passage mentions that even though the patient faces problems due to loss of verbal ability, he/she can often understand and return emotional signals. Option 4 contradicts this statement.
Hence, the correct answer is option 4.

CAT Practice Test - 17 - Question 39

The passage given below is followed by a questios. Choose the most appropriate answer to each question.

In far-off Syria, a country lying northeast of Palestine, the land in which Jesus was born, the farmers who keep vineyards are very much troubled with foxes and bears, which destroy their crops at night. And so, to protect their vineyards, they build high stone-walls about them, and put broken bottles on the top to keep these animals out, much as some people in this country who have orchards do, in order to keep out small boys. These fences keep out the bears, because they cut themselves on the glass in trying to climb over, and they also keep out some of the foxes. But after all, when the grapes are nearly ripe, the owners of the vineyards and their men are obliged to build platforms up above the trellises, and stay there all night, in order to guard their crops. These watchers manage very well with all the other wild animals excepting the little foxes. They can see the big foxes and drive them off, but the little ones they cannot see, and so these destroy the vines. I suppose that it was an experience something like that which led one of the Bible-writers to say that the little foxes destroy the vines. It seems to me that this is very true with sins, too; it is the little sins that destroy us. When a big sin like stealing, lying or cheating comes along we can see that easily enough, and we will not let it over the fence into our lives. We drive it away, and are soon rid of it. But when the little sins come, like little foxes, we do not see them, and so they get in and destroy our character. What are some of these little foxes? I think one is pride, which makes you so conceited, because you live in a big house or have an automobile or fine clothes, that you will not speak to or play with other boys and girls who have not quite such fine things, although they may be just as bright and just as good as you. Pride is a little fox that kills the vine of brotherliness which Christ planted in our hearts. Then another little fox is sulkiness. Sulkiness makes you frown and go away in a corner. It sucks up all the sunlight there is, and makes the world very gray and dull, like a day in November. This fox kills the vine called “peace” which Christ planted.
One more little fox is jealousy. This makes boys and girls dislike others who get higher marks than they in school, or who have more friends, or better toys. It is one of the most destructive little foxes there is, for it kills the best vine of all that Christ planted: that is, love. Be careful, then, boys and girls, of these little foxes, for they are worse than bears and big foxes, because they look so small and harmless, and slip by when you are not paying attention, but which destroy your character as readily as the others.

 

Q.Which of the following cannot be inferred from the passage?

Detailed Solution for CAT Practice Test - 17 - Question 39

From the passage, “These watchers manage very well with all the other wild animals excepting the little foxes. They can see the big foxes and drive them off, but the little ones they cannot see, and so these destroy the vines”. Hence, high stone walls do not keep out the “little foxes”. Hence, option 1 can be inferred. Similarly, it can be inferred from the above that the farmers cannot see the “little foxes” because of their small size while they can spot the “big foxes”. Hence, option 2 is incorrect.
Option 4 can be inferred from the following five extracts, “Pride is a little fox that kills the vine of brotherliness which Christ planted in our hearts”. “Then another little fox is sulkiness” combined with “This fox kills the vine called “peace” which Christ planted” and “One more little fox is jealousy” combined with “It is one of the most destructive little foxes there is, for it kills the best vine of all that Christ planted: that is, love”.
Little foxes are compared with little sins in the above quote, not ‘all the sins’. Hence, option 3 cannot be inferred.
Hence, the correct answer is option 3.

CAT Practice Test - 17 - Question 40

The passage given below is followed by a questios. Choose the most appropriate answer to each question.

In far-off Syria, a country lying northeast of Palestine, the land in which Jesus was born, the farmers who keep vineyards are very much troubled with foxes and bears, which destroy their crops at night. And so, to protect their vineyards, they build high stone-walls about them, and put broken bottles on the top to keep these animals out, much as some people in this country who have orchards do, in order to keep out small boys. These fences keep out the bears, because they cut themselves on the glass in trying to climb over, and they also keep out some of the foxes. But after all, when the grapes are nearly ripe, the owners of the vineyards and their men are obliged to build platforms up above the trellises, and stay there all night, in order to guard their crops. These watchers manage very well with all the other wild animals excepting the little foxes. They can see the big foxes and drive them off, but the little ones they cannot see, and so these destroy the vines. I suppose that it was an experience something like that which led one of the Bible-writers to say that the little foxes destroy the vines. It seems to me that this is very true with sins, too; it is the little sins that destroy us. When a big sin like stealing, lying or cheating comes along we can see that easily enough, and we will not let it over the fence into our lives. We drive it away, and are soon rid of it. But when the little sins come, like little foxes, we do not see them, and so they get in and destroy our character. What are some of these little foxes? I think one is pride, which makes you so conceited, because you live in a big house or have an automobile or fine clothes, that you will not speak to or play with other boys and girls who have not quite such fine things, although they may be just as bright and just as good as you. Pride is a little fox that kills the vine of brotherliness which Christ planted in our hearts. Then another little fox is sulkiness. Sulkiness makes you frown and go away in a corner. It sucks up all the sunlight there is, and makes the world very gray and dull, like a day in November. This fox kills the vine called “peace” which Christ planted.
One more little fox is jealousy. This makes boys and girls dislike others who get higher marks than they in school, or who have more friends, or better toys. It is one of the most destructive little foxes there is, for it kills the best vine of all that Christ planted: that is, love. Be careful, then, boys and girls, of these little foxes, for they are worse than bears and big foxes, because they look so small and harmless, and slip by when you are not paying attention, but which destroy your character as readily as the others.

 

Q.Why are the owners of the vineyards obliged to build platforms and stay there all night when they have already erected high fences? 

Detailed Solution for CAT Practice Test - 17 - Question 40

Option 1 is correct. From the passage, “...when the grapes are nearly ripe, the owners of the vineyards and their men are obliged to build platforms up above the trellises, and stay there all night, in order to guard their crops.” Option 2 may be correct, but does not answer the question asked. Similarly, options 3 and 4 are partially correct, but do not provide the complete answer.
Hence, the correct answer is option 1.

CAT Practice Test - 17 - Question 41

The passage given below is followed by a questios. Choose the most appropriate answer to each question.

In far-off Syria, a country lying northeast of Palestine, the land in which Jesus was born, the farmers who keep vineyards are very much troubled with foxes and bears, which destroy their crops at night. And so, to protect their vineyards, they build high stone-walls about them, and put broken bottles on the top to keep these animals out, much as some people in this country who have orchards do, in order to keep out small boys. These fences keep out the bears, because they cut themselves on the glass in trying to climb over, and they also keep out some of the foxes. But after all, when the grapes are nearly ripe, the owners of the vineyards and their men are obliged to build platforms up above the trellises, and stay there all night, in order to guard their crops. These watchers manage very well with all the other wild animals excepting the little foxes. They can see the big foxes and drive them off, but the little ones they cannot see, and so these destroy the vines. I suppose that it was an experience something like that which led one of the Bible-writers to say that the little foxes destroy the vines. It seems to me that this is very true with sins, too; it is the little sins that destroy us. When a big sin like stealing, lying or cheating comes along we can see that easily enough, and we will not let it over the fence into our lives. We drive it away, and are soon rid of it. But when the little sins come, like little foxes, we do not see them, and so they get in and destroy our character. What are some of these little foxes? I think one is pride, which makes you so conceited, because you live in a big house or have an automobile or fine clothes, that you will not speak to or play with other boys and girls who have not quite such fine things, although they may be just as bright and just as good as you. Pride is a little fox that kills the vine of brotherliness which Christ planted in our hearts. Then another little fox is sulkiness. Sulkiness makes you frown and go away in a corner. It sucks up all the sunlight there is, and makes the world very gray and dull, like a day in November. This fox kills the vine called “peace” which Christ planted.
One more little fox is jealousy. This makes boys and girls dislike others who get higher marks than they in school, or who have more friends, or better toys. It is one of the most destructive little foxes there is, for it kills the best vine of all that Christ planted: that is, love. Be careful, then, boys and girls, of these little foxes, for they are worse than bears and big foxes, because they look so small and harmless, and slip by when you are not paying attention, but which destroy your character as readily as the others.

 

Q.The author has compared little foxes to small sins because: 

Detailed Solution for CAT Practice Test - 17 - Question 41

Options 1 is partially correct. The author conveys that small sins are more dangerous than big sins because we can recognize and avoid big sins but are not aware of little sins that we commit. Option 2 best conveys this.
Option 3, on the other hand, reverses what is said in the passage by asking to ignore the idea which is opposite to the core idea. Option 4 is not mentioned in the passage.
Hence, the correct answer is option 2.

CAT Practice Test - 17 - Question 42

The passage given below is followed by a questios. Choose the most appropriate answer to each question.

In far-off Syria, a country lying northeast of Palestine, the land in which Jesus was born, the farmers who keep vineyards are very much troubled with foxes and bears, which destroy their crops at night. And so, to protect their vineyards, they build high stone-walls about them, and put broken bottles on the top to keep these animals out, much as some people in this country who have orchards do, in order to keep out small boys. These fences keep out the bears, because they cut themselves on the glass in trying to climb over, and they also keep out some of the foxes. But after all, when the grapes are nearly ripe, the owners of the vineyards and their men are obliged to build platforms up above the trellises, and stay there all night, in order to guard their crops. These watchers manage very well with all the other wild animals excepting the little foxes. They can see the big foxes and drive them off, but the little ones they cannot see, and so these destroy the vines. I suppose that it was an experience something like that which led one of the Bible-writers to say that the little foxes destroy the vines. It seems to me that this is very true with sins, too; it is the little sins that destroy us. When a big sin like stealing, lying or cheating comes along we can see that easily enough, and we will not let it over the fence into our lives. We drive it away, and are soon rid of it. But when the little sins come, like little foxes, we do not see them, and so they get in and destroy our character. What are some of these little foxes? I think one is pride, which makes you so conceited, because you live in a big house or have an automobile or fine clothes, that you will not speak to or play with other boys and girls who have not quite such fine things, although they may be just as bright and just as good as you. Pride is a little fox that kills the vine of brotherliness which Christ planted in our hearts. Then another little fox is sulkiness. Sulkiness makes you frown and go away in a corner. It sucks up all the sunlight there is, and makes the world very gray and dull, like a day in November. This fox kills the vine called “peace” which Christ planted.
One more little fox is jealousy. This makes boys and girls dislike others who get higher marks than they in school, or who have more friends, or better toys. It is one of the most destructive little foxes there is, for it kills the best vine of all that Christ planted: that is, love. Be careful, then, boys and girls, of these little foxes, for they are worse than bears and big foxes, because they look so small and harmless, and slip by when you are not paying attention, but which destroy your character as readily as the others.

 

Q.Why has the author compared the little foxes to three sins, namely pride, sulkiness and jealousy?

Detailed Solution for CAT Practice Test - 17 - Question 42

Options 1 and 2 are not mentioned in the passage, nor can they be implied in any way.
Option 4 does not answer the question stem in a logical or coherent manner.
Option 3 is the correct answer option since it answers the question directly as to why the little foxes are being compared to these three sins, namely that just as the little foxes are more dangerous than the big foxes because they cannot be detected while they are climbing over the fences so also these small sins are more dangerous for us than big sins because similarly, they too are difficult to detect.
Hence, the correct answer is option 3.

CAT Practice Test - 17 - Question 43

The passage given below is followed by a questios. Choose the most appropriate answer to each question.

In far-off Syria, a country lying northeast of Palestine, the land in which Jesus was born, the farmers who keep vineyards are very much troubled with foxes and bears, which destroy their crops at night. And so, to protect their vineyards, they build high stone-walls about them, and put broken bottles on the top to keep these animals out, much as some people in this country who have orchards do, in order to keep out small boys. These fences keep out the bears, because they cut themselves on the glass in trying to climb over, and they also keep out some of the foxes. But after all, when the grapes are nearly ripe, the owners of the vineyards and their men are obliged to build platforms up above the trellises, and stay there all night, in order to guard their crops. These watchers manage very well with all the other wild animals excepting the little foxes. They can see the big foxes and drive them off, but the little ones they cannot see, and so these destroy the vines. I suppose that it was an experience something like that which led one of the Bible-writers to say that the little foxes destroy the vines. It seems to me that this is very true with sins, too; it is the little sins that destroy us. When a big sin like stealing, lying or cheating comes along we can see that easily enough, and we will not let it over the fence into our lives. We drive it away, and are soon rid of it. But when the little sins come, like little foxes, we do not see them, and so they get in and destroy our character. What are some of these little foxes? I think one is pride, which makes you so conceited, because you live in a big house or have an automobile or fine clothes, that you will not speak to or play with other boys and girls who have not quite such fine things, although they may be just as bright and just as good as you. Pride is a little fox that kills the vine of brotherliness which Christ planted in our hearts. Then another little fox is sulkiness. Sulkiness makes you frown and go away in a corner. It sucks up all the sunlight there is, and makes the world very gray and dull, like a day in November. This fox kills the vine called “peace” which Christ planted.
One more little fox is jealousy. This makes boys and girls dislike others who get higher marks than they in school, or who have more friends, or better toys. It is one of the most destructive little foxes there is, for it kills the best vine of all that Christ planted: that is, love. Be careful, then, boys and girls, of these little foxes, for they are worse than bears and big foxes, because they look so small and harmless, and slip by when you are not paying attention, but which destroy your character as readily as the others.

 

Q.The manner in which “little foxes” are compared to “little sins” most closely corresponds with which of the following?

Detailed Solution for CAT Practice Test - 17 - Question 43

Euphemism means ‘the substitution of a mild, indirect, or vague expression for one thought to be offensive, harsh, or blunt.’ For example, “He passed away” instead of “He died”.
Oxymoron refers to the combination of incongruous or contradictory terms. For example, “He was cruelly kind to me”. Tautology refers to the needless repetition of an idea. For example, “She will decide whether to go to the market or not go to the market”. ‘Allegory’ means ‘a representation of an abstract or spiritual meaning through concrete or material forms’.
From the meanings, allegory best fits the comparison - foxes (material) are being compared to sins (abstract).
Hence, the correct answer is option 4.

CAT Practice Test - 17 - Question 44

The passage given below is followed by a questios. Choose the most appropriate answer to each question.

In far-off Syria, a country lying northeast of Palestine, the land in which Jesus was born, the farmers who keep vineyards are very much troubled with foxes and bears, which destroy their crops at night. And so, to protect their vineyards, they build high stone-walls about them, and put broken bottles on the top to keep these animals out, much as some people in this country who have orchards do, in order to keep out small boys. These fences keep out the bears, because they cut themselves on the glass in trying to climb over, and they also keep out some of the foxes. But after all, when the grapes are nearly ripe, the owners of the vineyards and their men are obliged to build platforms up above the trellises, and stay there all night, in order to guard their crops. These watchers manage very well with all the other wild animals excepting the little foxes. They can see the big foxes and drive them off, but the little ones they cannot see, and so these destroy the vines. I suppose that it was an experience something like that which led one of the Bible-writers to say that the little foxes destroy the vines. It seems to me that this is very true with sins, too; it is the little sins that destroy us. When a big sin like stealing, lying or cheating comes along we can see that easily enough, and we will not let it over the fence into our lives. We drive it away, and are soon rid of it. But when the little sins come, like little foxes, we do not see them, and so they get in and destroy our character. What are some of these little foxes? I think one is pride, which makes you so conceited, because you live in a big house or have an automobile or fine clothes, that you will not speak to or play with other boys and girls who have not quite such fine things, although they may be just as bright and just as good as you. Pride is a little fox that kills the vine of brotherliness which Christ planted in our hearts. Then another little fox is sulkiness. Sulkiness makes you frown and go away in a corner. It sucks up all the sunlight there is, and makes the world very gray and dull, like a day in November. This fox kills the vine called “peace” which Christ planted.
One more little fox is jealousy. This makes boys and girls dislike others who get higher marks than they in school, or who have more friends, or better toys. It is one of the most destructive little foxes there is, for it kills the best vine of all that Christ planted: that is, love. Be careful, then, boys and girls, of these little foxes, for they are worse than bears and big foxes, because they look so small and harmless, and slip by when you are not paying attention, but which destroy your character as readily as the others.

 

Q.The author of the passage calls jealousy as the most destructive sins of all because 

Detailed Solution for CAT Practice Test - 17 - Question 44

Option 1 is incorrect as the passage says "jealousy" kills the best vine of all that Christ planted. Hence, one cannot infer it it is the best vine of the vineyard.
Option 3 is incorrect as the passage says love is the best vine of all that Christ planted and not that it is closest to the Christ. Option 4 is incorrect as it does not clearly say the emotion underlying the relation between the boys and girls.
Option 2 aptly answers the question as it can be inferred from the line “best vine of all that Christ planted: that is, love” that the emotion “love” which is deemed to be the best vines of all that planted by Christ(sacred emotion) is attacked by jealousy and so "jealousy" is called destructive.
Hence, the correct answer is option 2.

CAT Practice Test - 17 - Question 45

The three tables below show certain details about the mobile phone market of a country.


A bank offers an EMI option for one year (at S.l.) to cover 30% of the mobile cost. The table below shows the monthly EMI to be paid for mobiles in categories B and C.

Q. Which company made the most expensive mobile in category B in 2011 ?uwie

Detailed Solution for CAT Practice Test - 17 - Question 45

30% of the total cost of the mobile is paid through EMIs.
EMI paid per month x 12 = Total amount paid in the year Total amount paid in the year = (1 + r%)(Total principal paid through EMI) EMI x 12 = (1 + r%) (30% of total cost of mobile) Since only the most expensive mobile is to be identified,
only the maximum value of needs to be found.
The mobile with maximum value of this ratio will also have the highest total cost.
Micromad: 
Huwei:
Telcom:
Fenovo: 
Thus, Huwei is the costliest mobile in category B in 2011. Hence, option 2.

CAT Practice Test - 17 - Question 46

The three tables below show certain details about the mobile phone market of a country.


A bank offers an EMI option for one year (at S.l.) to cover 30% of the mobile cost. The table below shows the monthly EMI to be paid for mobiles in categories B and C.


Q.From 2009 to 2011, the sales (by volume) of which category of mobiles have shown the highest percentage growth?

Detailed Solution for CAT Practice Test - 17 - Question 46

Correct Answer :- c

Explanation : From 2009 to 2011

Category A = 1 * 0.9 * 1 = 0.9 

Category B = 1 * 1 x 1.1 = 1.1

Category C = 1 * 0.8 * 0.9 = 0.72

Category D = 1 * 1.1 * 1.2 = 1.32

Thus, D has the highest growth rate.

CAT Practice Test - 17 - Question 47

The three tables below show certain details about the mobile phone market of a country.



A bank offers an EMI option for one year (at S.l.) to cover 30% of the mobile cost. The table below shows the monthly EMI to be paid for mobiles in categories B and C.


Q.If, in category D, only the given companies make mobiles, how many mobiles were sold in category D in 2010?

Detailed Solution for CAT Practice Test - 17 - Question 47

Total mobiles sold in category D in 2011 = 3000 + 2500 + 2000 +3300 = 10800
∴ Number of mobiles sold in category D in 2010 
Hence, option 2.

*Answer can only contain numeric values
CAT Practice Test - 17 - Question 48

Answer the following question based on the information given below.
In a certain training institute which had exactly 200 students, each student was given exactly one question from one among the following categories: Reasoning, Mathematics, Grammar, Interpretation and Comprehension.

Chart 1 gives the distribution of questions among the students.
However, since the students were not happy with the type of questions each one had received, they exchanged their questions with other students in such a way that none of them had a question of the same category as they had earlier.

Chart 2 shows the distribution of number of students who initially had questions from Reasoning category and exchanged their questions across students of various categories.

Further, exactly 8 students who originally had questions on Interpretation now have questions on Reasoning with them. Also 8 students who initially had questions on comprehension now have questions on Interpretation category.

Also out of the students who initially had questions on Grammar, 3 students now have questions on Reasoning and 16 have questions on Mathematics category.

 

Q.Out of the students who initially had questions on Interpretation,if the number of students who now have questions on Mathematics and Grammar is the same, then at least how many of the students who now have questions on Mathematics initially had questions on Comprehension?


Detailed Solution for CAT Practice Test - 17 - Question 48

The students exchanged questions among themselves, number of students having questions from each category will remain the same as before exchange.
Let the number of students who initially had questions on Mathematics and have questions on Reasoning after exchange be a.
Let the number of students who initially had questions on Mathematics and after exchange have questions on Grammar be b.
Let the number of students who initially had questions on Grammar and after exchange have questions on Interpretation be c.
Let the number of students who initially had questions on Interpretation and after exchange have questions on Mathematics be d.

Tabulating the given data we have the table:


R = Reasoning M = Mathematics G = Grammer I = Interpretation C = Comprehension Out of the students who initially had questions on Interpretation, the number of students having questions on Mathematics is d and the number of students having questions on Grammar is I I - a - b - d. v These two numbers are equal, we get, 11 - a - b - d = d 2af + a + b = 11 ••• We are supposed to minimize the number of students who now have questions on Mathematics and initially had questions on Comprehension, i.e. (15 - d), we need to maximize the value of of.
Now maximum possible value of d is 5, when a + b = 1 since a and b cannot have negative values.
The minimum possible value of (15 - d) is 10.
Answer: 10

*Answer can only contain numeric values
CAT Practice Test - 17 - Question 49

In a certain training institute which had exactly 200 students, each student was given exactly one question from one among the following categories: Reasoning, Mathematics, Grammar, Interpretation and Comprehension.

Chart 1 gives the distribution of questions among the students.
However, since the students were not happy with the type of questions each one had received, they exchanged their questions with other students in such a way that none of them had a question of the same category as they had earlier.

Chart 2 shows the distribution of number of students who initially had questions from Reasoning category and exchanged their questions across students of various categories.

Further, exactly 8 students who originally had questions on Interpretation now have questions on Reasoning with them. Also 8 students who initially had questions on comprehension now have questions on Interpretation category.

Also out of the students who initially had questions on Grammar, 3 students now have questions on Reasoning and 16 have questions on Mathematics category.

 

Q.Out of the students who initially had questions on Comprehension, what is the minimum number of students who now have questions on Reasoning category?


Detailed Solution for CAT Practice Test - 17 - Question 49

Out of the students who initially had questions on Comprehension category, the number of students who now have questions from Reasoning category is 32 - a.
Now maximum possible value of a is 11 from (11 - a - b - d), when b = d = 0 [v (11 - a - b - d), a, b and d are non negative integers].
(Refer the table given in the solution of the first question of the set).
The minimum number of required students will be 32 - 11 =21 
Answer: 21

*Answer can only contain numeric values
CAT Practice Test - 17 - Question 50

In a certain training institute which had exactly 200 students, each student was given exactly one question from one among the following categories: Reasoning, Mathematics, Grammar, Interpretation and Comprehension.

Chart 1 gives the distribution of questions among the students.
However, since the students were not happy with the type of questions each one had received, they exchanged their questions with other students in such a way that none of them had a question of the same category as they had earlier.

Chart 2 shows the distribution of number of students who initially had questions from Reasoning category and exchanged their questions across students of various categories.

Further, exactly 8 students who originally had questions on Interpretation now have questions on Reasoning with them. Also 8 students who initially had questions on comprehension now have questions on Interpretation category.

Also out of the students who initially had questions on Grammar, 3 students now have questions on Reasoning and 16 have questions on Mathematics category.

 

If out of the students who initially had questions on Mathematics with them, the number of students who now have questions on Comprehension is the maximum possible; then out of the students who now have questions on Grammar category what is the maximum possible number of students who initially had questions on Comprehension category?


Detailed Solution for CAT Practice Test - 17 - Question 50

Out of the students who initially had questions on Mathematics with them, the number of students who now have questions on Comprehension category is (29 + c - a - b).
This would be maximum when a and b are minimum possible, i.e. equal to 0 and further the value of c should be maximum possible.
The maximum possible value of c can be 8 from (8 - c).
The maximum possible value of (29 + c - a - b) can be 29 + 8 = 37
From among the students who now have questions on Grammar category the number of students who initially had question on Comprehension category is (5 + a + d).
Now, the maximum possible value of d can be 11 from (11 - a - b - d).'

• The required answer would be (5 + 0 + 11) = 16 Answer: 

*Answer can only contain numeric values
CAT Practice Test - 17 - Question 51

In a certain training institute which had exactly 200 students, each student was given exactly one question from one among the following categories: Reasoning, Mathematics, Grammar, Interpretation and Comprehension.

Chart 1 gives the distribution of questions among the students.
However, since the students were not happy with the type of questions each one had received, they exchanged their questions with other students in such a way that none of them had a question of the same category as they had earlier.

Chart 2 shows the distribution of number of students who initially had questions from Reasoning category and exchanged their questions across students of various categories.

Further, exactly 8 students who originally had questions on Interpretation now have questions on Reasoning with them. Also 8 students who initially had questions on comprehension now have questions on Interpretation category.

Also out of the students who initially had questions on Grammar, 3 students now have questions on Reasoning and 16 have questions on Mathematics category.

 

Q.If out of the students who initially had questions on Mathematics,  the number of students who now have questions on Reasoning is twice the number of students now having questions on Grammar, then at least how many students now have questions on Comprehension with them who initially had questions on Mathematics with them?


Detailed Solution for CAT Practice Test - 17 - Question 51

Out of the students who initially had questions from Mathematics category, the number of students who now have questions on Reasoning is twice the number of students now having questions on Grammar. a - 2 b
Number of students now having question on Comprehension with them who initially had questions on Mathematics is (29 + c - a - b). v a = 2b, the expresssion would reduce to 29 + c - 3b Now, (29 + c - 3b) would be minimum when c = 0 and b is maximum possible.
Now, the maximum possible value of (a + b) is 11 from (11 - a - b - d), or maximum possible value of 2b + b [for a = 2b] i.e. 3b is 11.
The maximum possible value of b = 3 The minimum possible value of (29 + c - 3b) will be 20.
Answer: 20

CAT Practice Test - 17 - Question 52

Answer the following question based on the information given below.

In a student exchange program, four prominent management institutes from the city had shown an initiative to facilitate learning for their students.
The institutes were: The Pride, The Mascot, The Embezzlers and The Educators.
The selected destinations for each of these institutes were USA, Japan, France, Australia, Germany, Korea and Canada.
The chart below gives the breakup of students sent by each of these institutes to each of these countries in terms of percentage of the total students sent by it to all the places put together.
Additional Information: Canada had exactly same number of students from two of the institutes.

 

Q.The total number of students sent by The Pride was more than sent Embezzlers as well as that by The Educators.
What can be said regarding the following statements?

1. Australia had more students from The Pride than from The Educators.

2. The number of students that had been to France from The Embezzlers and The Educators taken together was less than the number of students that had been to France from The Pride.

Detailed Solution for CAT Practice Test - 17 - Question 52

Let the total number of students sent by The Pride, The Mascot, The Embezzlers and The Educators be 100a, 100b, 100c, 100d respectively.
From the conditions given, we have, a > c and a > d 12% of the students sent by The Pride and 20% of the students sent by The Educators went to Australia.
Thus, Statement 1 means 12a > 20of, which may or may not be true.
Similarly, 21% of the students sent by The Pride, 11% of the students sent by The Embezzlers and 10% of the students sent by The Educators went to France.
Thus, statement2 means, 11c+10of<21a We know that c < a and d < a a 11 c < 1 1 a and 10 o f < 10a 11c + 10d< 11a + 10a i.e. 11c + 10d< 21a Thus statement 2 is definitely true.
Thus, only one statement is definitely true.
Also, the two statements are independent of each other and one being true has no impact on the other.
Hence, option 2.

CAT Practice Test - 17 - Question 53

In a student exchange program, four prominent management institutes from the city had shown an initiative to facilitate learning for their students.
The institutes were: The Pride, The Mascot, The Embezzlers and The Educators.
The selected destinations for each of these institutes were USA, Japan, France, Australia, Germany, Korea and Canada.
The chart below gives the breakup of students sent by each of these institutes to each of these countries in terms of percentage of the total students sent by it to all the places put together.
Additional Information: Canada had exactly same number of students from two of the institutes.

 

Q.What can be said regarding the following two statements? 
1. Germany had witnessed an equal number of students from three institutes.

2. Canada had witnessed an equal number of students from The Mascot and The Embezzlers.

Detailed Solution for CAT Practice Test - 17 - Question 53

Since statement 2 provides more specific data compared to statement 1 and since 2 answer options assume statement 2 to be true, initially consider statement 2 to be true.
Let the total number of students sent abroad by The Pride, The Mascot, The Embezzlers and The Educators be 100a, 100b, 100c and 100 d respectively.
Since statement 2 is true, you get 8b = 14c.
Now, the number of students in Germany from the four colleges is 25a, 16b, 28c and 11 d respectively.
Since 8b = 14c, 16b = 28c.
Thus, The Mascot and The Embezzlers were two colleges that sent an equal number of students to Germany.
Now, it is not known whether 25a or 11dor both are equal to 16b and 28c.
If any one of them is equal, there will be exactly three colleges with equal number of students in Germany. In such a case, statement 1 will become definitely true.
If neither of them is equal to 16b and 28c, there will 2 colleges with equal students and if both are equal, there will be 4 colleges with equal number of students. In both these cases, statement 1 will become definitely false.
Thus, if statement 2 is true, statement 1 may be true or false, depending on the value of 25a and 11 d.
Hence, options 2 and 3 can be eliminated.

Now, assume that statement 1 is true.
Since there are 4 colleges and three colleges sent an equal number of students to Germany, there are 4 combinations possible.
If The Mascot, The Embezzlers and any one of the other two colleges sent an equal number of students to Germany, then the relationship becomes either 25a = 16b = 28c or 16b = 28c = 11 of In both these cases, 8b= 14c and so, statement 2 becomes definitely true.
Since statement 2 can be true in some cases, option 1 can also be eliminated.
Hence, option 4.

CAT Practice Test - 17 - Question 54

In a student exchange program, four prominent management institutes from the city had shown an initiative to facilitate learning for their students.
The institutes were: The Pride, The Mascot, The Embezzlers and The Educators.
The selected destinations for each of these institutes were USA, Japan, France, Australia, Germany, Korea and Canada.
The chart below gives the breakup of students sent by each of these institutes to each of these countries in terms of percentage of the total students sent by it to all the places put together.
Additional Information: Canada had exactly same number of students from two of the institutes.

 

Q.Which of the following is definitely true?

Detailed Solution for CAT Practice Test - 17 - Question 54

Let the total number of students sent abroad by The Pride, The Mascot, The Embezzlers and The Educators be 100a, 1005, 100c and 100cf respectively. Statement 1 can be represented as 10a + 20c > 21a + 11c, implying that 9c > 11a, i.e. c > a.
The number of students who went to Korea from The Pride and The Embezzlers can be represented as 15a and 15c respectively. v c > a 15c > 15a Thus statement 1 is true.
Hence, option 4 can be eliminated.
Statement 2 can be represented as 10a + 20c >12a + 7c, implying that 13c > 2a.
Here, c may or may not be greater than a.
Thus, the number of students who went abroad from The Pride may or may not greater than the number of students who went abroad from The Embezzlers.
Therefore, statement 2 may or may not be true.
Hence, option 1.

CAT Practice Test - 17 - Question 55

In a student exchange program, four prominent management institutes from the city had shown an initiative to facilitate learning for their students.
The institutes were: The Pride, The Mascot, The Embezzlers and The Educators.
The selected destinations for each of these institutes were USA, Japan, France, Australia, Germany, Korea and Canada.
The chart below gives the breakup of students sent by each of these institutes to each of these countries in terms of percentage of the total students sent by it to all the places put together.
Additional Information: Canada had exactly same number of students from two of the institutes.

 

Q.If Canada received the same number of students from The Pride and The Embezzlers, and USA had an equal number of students from The Mascot and The Educators, then which of the following statements is/are false? 

Detailed Solution for CAT Practice Test - 17 - Question 55

Let the total number of students sent abroad from The Pride, The Mascot, The Embezzlers and The Educators be 100a, 100b, 100c and 100 d respectively.
From the given conditions, we have 7a = 14c ... (i) and 8b = 11 d ... (ii) From condition (ii) b > d Also, a = 2c Express the statement in each option in terms of an inequation. Option 1: 22b > 20of Since b > d, the given relationship is correct.
Thus, the statement in option 1 is true.
Option 2: 10a = 20c Since a = 2c, the given relationship is correct.
Thus, the statement in option 2 is true.
Option 3: 10b < 13cf b > o f such that 8b = 11 d Thus, b = (11/8)</ 10b = 13.75c/ 13.75c/> 13c/ 10b > 13c/ Thus, the statement in option 3 is false.
Hence, option 3.

CAT Practice Test - 17 - Question 56

In a student exchange program, four prominent management institutes from the city had shown an initiative to facilitate learning for their students.
The institutes were: The Pride, The Mascot, The Embezzlers and The Educators.
The selected destinations for each of these institutes were USA, Japan, France, Australia, Germany, Korea and Canada.
The chart below gives the breakup of students sent by each of these institutes to each of these countries in terms of percentage of the total students sent by it to all the places put together.
Additional Information: Canada had exactly same number of students from two of the institutes.

 

Q.If The Mascot sends the maximum students to Japan, what is the minimum number of countries, other than Japan, which also definitely have maximum students from The Mascot? 

Detailed Solution for CAT Practice Test - 17 - Question 56

Let the number of students who have gone abroad from The Pride, The Mascot, The Embezzlers and The Educators be 100a, 100b, 100c and 100d respectively.
The Mascot send the maximum number of students to Japan. 21b > 10a, 21b > 20c and 21b > 27d The only conclusion from this is that b > d.
Since b cannot be compared with a and c, there is no country where it can be definitely said that The Mascot sends the maximum number of students.
Hence, option 1.

CAT Practice Test - 17 - Question 57

Answer the following question based on the information given below.

The table given below shows the average marks obtained by IIT and Non IIT (students who are not from IIT) students in three sections “Quantitative Ability”, “Data Interpretation” and “Verbal Ability” of seven different iCATs 01 to 07. The breakup of IIT and Non IIT students that took these seven iCATs was different from test to test, but across all the seven iCATs, the total number of students that took the test remained static at 100. Each iCAT had only three sections and each section of each iCAT had 25 questions. Each question carried 1 mark and there were no negative marks.

 

Q.For how many iCATs can the average total (all sections taken together) be computed for all the students (IIT and Non IIT) combined?

Detailed Solution for CAT Practice Test - 17 - Question 57

The total number of students who took each iCAT is known as 100.
But the number of IIT students and Non IIT students that took these tests is not known.
The combined average of all the students across all three sections can only be calculated for those iCATs where the average marks obtained by an IIT student are equal to the average marks obtained by a non IIT student.
The average marks obtained by an IIT student and a non IIT student across all sections are as shown in the table below.

Let there be x IIT students and y non IIT students taking each iCAT such that x + y = 100 Consider iCAT 01.
The combined average = (34x + 34y)/(x + y) = 34 Consider iCAT 02.
The combined average = (34x + 32y)/(x + y). Since the value of x and y is not known, this value cannot be calculated.
The combined average can be computed only for 3 iCATs i.e. iCAT 01, 04 and 05.
Hence, option 1.

CAT Practice Test - 17 - Question 58

The table given below shows the average marks obtained by IIT and Non IIT (students who are not from IIT) students in three sections “Quantitative Ability”, “Data Interpretation” and “Verbal Ability” of seven different iCATs 01 to 07. The breakup of IIT and Non IIT students that took these seven iCATs was different from test to test, but across all the seven iCATs, the total number of students that took the test remained static at 100. Each iCAT had only three sections and each section of each iCAT had 25 questions. Each question carried 1 mark and there were no negative marks.

 

Q.For how many iCATs, is the average marks scored by IIT students in all the three sections combined definitely more than that of Non IIT?

Detailed Solution for CAT Practice Test - 17 - Question 58

In iCAT 02, 03, the average marks of IIT students is more than Non-IIT students.

Hence, option 3.

CAT Practice Test - 17 - Question 59

Answer the following question based on the information given below.

Rahul, who was preparing for his board exams, made a plan to tackle his weaker subjects - Physics, Chemistry and Biology - by devoting three hours in one sitting to any of the three subjects. To keep track of his progress, he had prepared a weekly schedule for himself along with a weekly test planned on a certain day of the week on which he did nothing except for writing the test and evaluating his performance. In his weekly schedule he had planned exactly three sittings for each of the three subjects. Also he ensured that the gap between any two consecutive sittings for any subject was not more than two days. Finally, this schedule was to be followed every week such that the condition related to the gap was never violated.

The table below gives the partial schedule of Rahul for 6 consecutive days.

Q. On which date did Rahul take his weekly exam in the above  period? He studied for all three subjects simultaneously on 13tn Feb.

Detailed Solution for CAT Practice Test - 17 - Question 59

Rahul studies each subject exactly thrice.
Also, the gap between two consecutive sittings of the same subject cannot be more than 2 days. This implies that, at the most, Rahul has to study a subject again every third day.
Now, note that there is atleast one subject mentioned per day for day 1 to day 6.
Since Rahul does not study any subject on the day he takes the test, he could have taken the test only on day 7.

Now, Rahul studies Biology on day 1 and day 4. Therefore, the last day on which he can sit for the third sitting of Biology is day 7.
Since day 7 is reserved for the test, Rahul has to sit for Biology on either day 5 or day 6.
Rahul studies Chemistry on day 3 and day 5. Therefore, he can study Chemistry on day 6 or day 1.
However, note that if Rahul studies Chemistry on day 6, he will have to mandatorily study Chemistry on day 1 or day 2 of the next week to satisfy the condition of not more than two days between consecutive sittings. However, this will lead to a different schedule for the second week. This is not permitted as per the last condition.
Therefore, Rahul definitely studies Chemistry on day 1.
Using a similar logic, Rahul can study Physics on any one of days 3, 4 and 5.
Rahul studies for all three subjects on 13th Feb.
Rahul can simultaneously study for all three subjects only on day 5.
Thus, day 5 is 13th Feb.
Since Rahul takes the weekly exam on day 7, he takes it on 15th Feb.
Hence, option 3.

CAT Practice Test - 17 - Question 60

Rahul, who was preparing for his board exams, made a plan to tackle his weaker subjects - Physics, Chemistry and Biology - by devoting three hours in one sitting to any of the three subjects. To keep track of his progress, he had prepared a weekly schedule for himself along with a weekly test planned on a certain day of the week on which he did nothing except for writing the test and evaluating his performance. In his weekly schedule he had planned exactly three sittings for each of the three subjects. Also he ensured that the gap between any two consecutive sittings for any subject was not more than two days. Finally, this schedule was to be followed every week such that the condition related to the gap was never violated.

The table below gives the partial schedule of Rahul for 6 consecutive days.

Q.On which of these dates did Rahul definitely have a sitting on exactly two subjects? Assume data from the first question.

Detailed Solution for CAT Practice Test - 17 - Question 60

Rahul could have studied for all three subjects only on 13th Feb, i.e. on day 5.
Now, if he studied Physics on day 5, he would not need to study Physics on day 3 (11th Feb) or Biology on day 6 (14th Feb).
In such a case, he would study exactly one subject each on these two days.
On the other, Rahul would definitely study Biology and Chemistry on 9th Feb i.e. on day 1.
Hence, option 1.

CAT Practice Test - 17 - Question 61

Rahul, who was preparing for his board exams, made a plan to tackle his weaker subjects - Physics, Chemistry and Biology - by devoting three hours in one sitting to any of the three subjects. To keep track of his progress, he had prepared a weekly schedule for himself along with a weekly test planned on a certain day of the week on which he did nothing except for writing the test and evaluating his performance. In his weekly schedule he had planned exactly three sittings for each of the three subjects. Also he ensured that the gap between any two consecutive sittings for any subject was not more than two days. Finally, this schedule was to be followed every week such that the condition related to the gap was never violated.

The table below gives the partial schedule of Rahul for 6 consecutive days.


Q.Which subject did Rahul definitely take on 14th Feb in his schedule? Assume data from the first question. 

Detailed Solution for CAT Practice Test - 17 - Question 61

Using the data from the first question, it is clear that 14th Feb is day 6 of Rahul’s schedule.
Rahul definitely has a sitting of Physics on this day.
He definitely does not study Chemistry on this day and may or may not study Biology on this day.
Therefore, definitely takes only Physics on 14th Feb.
Hence, option 1.

CAT Practice Test - 17 - Question 62

Rahul, who was preparing for his board exams, made a plan to tackle his weaker subjects - Physics, Chemistry and Biology - by devoting three hours in one sitting to any of the three subjects. To keep track of his progress, he had prepared a weekly schedule for himself along with a weekly test planned on a certain day of the week on which he did nothing except for writing the test and evaluating his performance. In his weekly schedule he had planned exactly three sittings for each of the three subjects. Also he ensured that the gap between any two consecutive sittings for any subject was not more than two days. Finally, this schedule was to be followed every week such that the condition related to the gap was never violated.

The table below gives the partial schedule of Rahul for 6 consecutive days.


Q.On which of these dates, did Rahul definitely have studied Chemistry? Assume the data from the first question. 

Detailed Solution for CAT Practice Test - 17 - Question 62

From the solution to the first question of the set, Rahul sttudied Chemistry on 9th, 11th and 13th Feb.
Hence, option 1.

CAT Practice Test - 17 - Question 63

Answer the following question based on the information given below.

In a Rolo Polo tournament, three teams; Team A, Team B and Team C were in the fray for the final winners position. Each team consisted of two members, one of whom was labelled as Rolo and the second member as Polo.
The peculiar characteristic of all the Rolos was that they never lied whereas all the Polos always lied. The names of all the six participants are Aryan, Bryan, Cyan, Dyan, Evan and Friam. The conversation of all these participants with an outsider was as follows:

I. Aryan : Cyan is representing team B.
II. Bryan : None of Aryan, Cyan and Evan is from Team A.
III. Bryan : Bryan is from Team B.
IV. Dyan : At most one of Aryan, Bryan and Evan is Polo.
V. Evan : Friam and I are in different teams.
VI. Friam : Dyan is a Polo from Team B.
VII. Aryan : Bryan is from Team C.
VIII. Cyan : Evan and I are from the same team.

 

Q. Who among the following is from Team B? 
I. Bryan

II. Dyan

III. Friam

IV. Aryan

Detailed Solution for CAT Practice Test - 17 - Question 63

Consider the statement ‘Evan and I are from the same team’ made by Cyan.
If Cyan tells the truth then Evan and Cyan must be in the same team. As Cyan is telling the truth it implies that Evan should necessarily lie. But Evan’s statement is also true as Friam and Evan are in different teams.
It implies that Cyan lies and must be a Polo.
Now consider the statement made by Friam, ‘Dyan is a Polo from Team B’.
If Friam tells the truth then Dyan must be a Polo from Team B and must lie.

 The statement made by Dyan must be a lie, implying that more than one of Aryan, Cyan and Evan must be Polos.
Now all three of them cannot be Polos as we have assumed Dyan to be lying thereby is a Polo. And the total number of Polos are only three.
If Aryan lies then Aryan, Dyan and Cyan must be Polos. And Bryan, Evan and Friam must be Rolos.
Using statements (III), (VI), (VII) and (VIII), the following can be concluded.
Either Aryan or Cyan must be from Team A. But from the second statement, it becomes evident that neither Aryan nor Cyan is in Team A Aryan cannot be a Polo.
If Evan tells a lie, then Evan, Cyan and Dyan must be Polos and Bryan, Aryan and Friam must be Rolos. Aryan’s statement that Cyan is from Team B and is a Polo contradicts with the statement made by Friam who said that Dyan is a Polo from Team B.

Evan cannot be a Polo.
Friam must be lying and must be a Polo.
Consider the fourth statement; if Dyan lies then either Aryan and Evan or one of them should be a Polo. But it is not possible as Cyan, Friam and Dyan are Polos.
Dyan must be a Rolo.
Bryan must be Polo as Dyan is a Rolo and he would tell a true. Now we get,


Consider statements (I) and (VII), Cyan and Bryan are in Team B and C respectively.
Friam must be from Team A.
From statements (V) and (VIII), Friam and Evan are in different teams; also Evan and Cyan are in different teams.
Evan must be from Team C.
As Bryan’s statement is a lie, atleast one of Aryan, Cyan and Evan must be in Team A.
Aryan must be in Team A and Dyan must be in Team B.
The final arrangement would be:

From the given options only Dyan is from Team B. Hence, option 2.

CAT Practice Test - 17 - Question 64

In a Rolo Polo tournament, three teams; Team A, Team B and Team C were in the fray for the final winners position. Each team consisted of two members, one of whom was labelled as Rolo and the second member as Polo.
The peculiar characteristic of all the Rolos was that they never lied whereas all the Polos always lied. The names of all the six participants are Aryan, Bryan, Cyan, Dyan, Evan and Friam. The conversation of all these participants with an outsider was as follows:

I. Aryan : Cyan is representing team B.
II. Bryan : None of Aryan, Cyan and Evan is from Team A.
III. Bryan : Bryan is from Team B.
IV. Dyan : At most one of Aryan, Bryan and Evan is Polo.
V. Evan : Friam and I are in different teams.
VI. Friam : Dyan is a Polo from Team B.
VII. Aryan : Bryan is from Team C.
VIII. Cyan : Evan and I are from the same team.

 

Q.What can be said about the following two statements? 3 Marks
I. Friam is from Team A.
II. Evan is from Team C.

Detailed Solution for CAT Practice Test - 17 - Question 64

Referring the solution given in the first question of the set, we get that both the statements are true.
Hence, option 4.

CAT Practice Test - 17 - Question 65

Answer the following question based on the information given below.

There are eight people Aamir, Amit, Rajni, Ajay, Abhishek, Akshay, Vishnu and Rama. Among them, there are four groups formed with two friends in each group. It is known that Aamir is not the friend of Amit or Rajni or Ajay. Abhishek is not the friend of Aamir or Ajay or Akshay. Ajay is not the friend of Aamir or Amit or Rama or Vishnu. Rama is not the friend of Abhishek or Rajni or Vishnu.

 

Q. Who among the following cannot be each other’s friends? 

Detailed Solution for CAT Practice Test - 17 - Question 65

From the given conditions, the possible friends for each person can be found as shown below:

Aamir - AkshayA/ishnu/Rama

Amit - Rajni/Abhishek/Akshay/Vishnu/Rama Rajni - Amit/Ajay/Abhishek/Akshay/Vishnu Ajay - Rajni/Akshay

Abhishek - Amit/Rajni/Vishnu

Akshay Aamir/Amit/Rajni/Ajay/Vishnu/Rama

Vishnu - Aamir/Amit/Rajni/Abhishek/Akshay Rama - Aamir/Amit/Akshay

Note that these 8 people are divided into 4 groups of 2 people each.
Now, consider the person who has the least possible number of friends i.e. Ajay.
Ajay has only two possible friends i.e. Akshay and Rajni.
Therefore, if Akshay and Rajni are each other’s friends, they will form a group by themselves and Ajay will never be able to form a group.
Therefore, Akshay and Rajni can never be each other’s friends. Hence, option 4.

CAT Practice Test - 17 - Question 66

There are eight people Aamir, Amit, Rajni, Ajay, Abhishek, Akshay, Vishnu and Rama. Among them, there are four groups formed with two friends in each group. It is known that Aamir is not the friend of Amit or Rajni or Ajay. Abhishek is not the friend of Aamir or Ajay or Akshay. Ajay is not the friend of Aamir or Amit or Rama or Vishnu. Rama is not the friend of Abhishek or Rajni or Vishnu.

 

Q.Assume that Akshay and Rama formed a group, who among the following was with Amit?

Detailed Solution for CAT Practice Test - 17 - Question 66

If Akshay formed a group with Rama, Ajay had to be with Rajni.
So, Amir must have formed a group with Vishnu and hence Abhishek must be with Amit.
Hence, option 2.

CAT Practice Test - 17 - Question 67

There are eight people Aamir, Amit, Rajni, Ajay, Abhishek, Akshay, Vishnu and Rama. Among them, there are four groups formed with two friends in each group. It is known that Aamir is not the friend of Amit or Rajni or Ajay. Abhishek is not the friend of Aamir or Ajay or Akshay. Ajay is not the friend of Aamir or Amit or Rama or Vishnu. Rama is not the friend of Abhishek or Rajni or Vishnu.

 

Q.Who cannot be the friend of Amit?

Detailed Solution for CAT Practice Test - 17 - Question 67

From the answer to the first question of the set, Ajay and Aamir cannot be Amit's friend.
Hence, option 4.

CAT Practice Test - 17 - Question 68

There are eight people Aamir, Amit, Rajni, Ajay, Abhishek, Akshay, Vishnu and Rama. Among them, there are four groups formed with two friends in each group. It is known that Aamir is not the friend of Amit or Rajni or Ajay. Abhishek is not the friend of Aamir or Ajay or Akshay. Ajay is not the friend of Aamir or Amit or Rama or Vishnu. Rama is not the friend of Abhishek or Rajni or Vishnu.

 

Q.If Akshay was with Aamir, then who formed a group with Rajni?

Detailed Solution for CAT Practice Test - 17 - Question 68

Ajay has only two friends - Rajni and Akshay.
If Akshay and Amit formed a group, Rajni and Ajay must have formed one group.

Hence, option 1.

CAT Practice Test - 17 - Question 69

Answer the following question based on the information given below.

A cuboid is to be painted with red, blue and green colours.


Conditions:

(1) A surface has only one colour.

(2) Each of the three colors must be used.

(3) All six surfaces must be painted.

The cuboid is then cut by 5, 6 and 7 equally spaced planes parallel to xy, yz and xz planes respectively.

 

Q.What can be the maximum number of cubes that do not have ether blue or green colour on them? 

Detailed Solution for CAT Practice Test - 17 - Question 69

We have cut the cuboid by 8, 6 and 7 equally spaced planes parallel to xy, yz and xz planes respectively.
Hence, We have:

2 surfaces of the cuboid with (8x7) = 56 squares

2 surfaces of the cuboid with (8x6) = 48 squares

and remaining 2 surfaces of the cuboid with (7x6) = 42 squares

Total number of cubes formed = 8 x 7 x 6 = 336 In order to have maximum number of cubes without green or blue colour,

(i) one of the surfaces of cuboid with 42 squares must have painted with green and the other with blue. (Green and Blue should be used at least once.)

(ii) remaining 4 surfaces must be red.
Cubes having blue colour on them = 42 Cubes having green colour on them = 42 Cubes having blue and green on them = 0 Cubes without blue or green on them = 336 - 84 = 252 Hence, option 2.

CAT Practice Test - 17 - Question 70

A cuboid is to be painted with red, blue and green colours.

Conditions:

(1) A surface has only one colour.

(2) Each of the three colors must be used.

(3) All six surfaces must be painted.

The cuboid is then cut by 5, 6 and 7 equally spaced planes parallel to xy, yz and xz planes respectively.

 

Q.What is the minimum number of cubes with exactly two of the three colors on them?

Detailed Solution for CAT Practice Test - 17 - Question 70

In order to have minimum number of cubes with exactly two of the three colors on them, let’s paint the 2 adjacent surfaces with 42 and 48 squares with green and blue respectively and remaining 4 surfaces with red colour.
All cubes along the perimeter of green surface will have exactly two colors except for cubes at 2 corners = 2 2 - 2 = 20

All cubes along the perimeter of blue surface will have exactly two colors except for cubes at 2 corners = 24 - 2 = 22 But 4 cubes along the edge joining blue and green surfaces are common.
None of the remaining cubes can have only 2 colors

Minimum number of cubes with exactly two of the three colors on them = (20 + 22 - 4) = 38 
Hence, option 2.

*Answer can only contain numeric values
CAT Practice Test - 17 - Question 71

The maximum number of cubes that do not have red or/and green surface but has blue surface:


Detailed Solution for CAT Practice Test - 17 - Question 71

Paint one of the 2 surfaces with 42 squares with green and the other with red and remaining 4 surfaces with blue.
Total number of cubes that are painted = (8 x 7 x 6) - (6 x 5 x 4) = 336-120 = 216
Cubes having green paint on it = 42 Cubes having red paint on it = 42 Maximum number of cubes that are painted only blue = 216 - 84 = 132
Answer: 132

*Answer can only contain numeric values
CAT Practice Test - 17 - Question 72

What is the total number of cubes with none of blue, green or red colour on its surface?


Detailed Solution for CAT Practice Test - 17 - Question 72

The dimension of the cube (8 * 7 * 6).
We need to find the cubes those are not on the surface of the bigger cube.
The required number of cubes = (8 - 2 ) x (7 - 2) * (6 - 2) = 120
Answer: 120

*Answer can only contain numeric values
CAT Practice Test - 17 - Question 73

Anirudh wants to have his bath in a bathtub. There are two taps, one for hot water and one for cold water. Both the taps can individually fill the bathtub in 15 minutes. He opens both the taps simultaneously but after 6 minutes he closes the cold water tap. He lets the hot water tap run until the bathtub fills up. The temperature of the hot water is 50° and of the cold water is 25°. How many degrees is the temperature of the water in the bathtub? Assume that no heat is lost to the surrounding atmosphere.


Detailed Solution for CAT Practice Test - 17 - Question 73

The hot and cold water tap can individually fill the bathtub in 15 minutes. In 1 minute they can individually fill 1/15 of the bath tub.  In 6 minutes they can individually fill 6/15 of the bath tub.

As both the ta p s run for 6 minutes, the volume o f tan k th at is filled  
40% + 40%
80%
The rest 20% of the tank is filled by the hot water.
So 60% of the tank is filled by hot water and the rest 40% by cold water.
The final temperature of the bath tub will be determined by the ratio of hot water to cold water in the bath tub which is 3 : 2 in this case and the temperatures of hot and cold water.
Using alligation formula

We get x = 40 The temperature of the bathtub is 40°. Answer: 40

 

*Answer can only contain numeric values
CAT Practice Test - 17 - Question 74

If x, y, z are distinct positive integers such that x2 + y2 = z2( 1 + xy) then how many ordered pairs of x, y and z satisfy this equation?


Detailed Solution for CAT Practice Test - 17 - Question 74

x2 + y2 > (1 + xy) for distinct x and y.
Only for z2 > 1, x2 + y2 = z2(1 + xy) The equation is symmetric in x and y, so we can assume (without loss of generality) x = y

2x2 = z2(1 + x2) ... (i)

As x is positive integer, we can write x2 < x2 + 1

x2z2 < z2(x2 + 1) ... (ii)

Using (ii) in (i) 2x2 > x2z2

z2 < 2

z = 1 (as z is positive integer)

 x2 + y2 = (1 + xy) This is true only for x = y = 1 Thus, there is no pair of distinct positive integers that satisfy the given equation.
Answer: 0

 

CAT Practice Test - 17 - Question 75

A car is being driven, in a straight line and at a uniform speed, towards the base of a vertical tower. The top of the tower is observed from the car and, in the process, it takes 15 minutes for the angle of elevation to change from 45° to 60°. After how much more time (in minutes) will this car reach the base of the tower?

Detailed Solution for CAT Practice Test - 17 - Question 75

Let x be the distance between the later position of the car and the tower (i.e. when the angle of elevation was 60°).
Since the triangle formed (i.e. ΔABD) is a 30°-60°-90° triangle, we have,

heigh t o f the tower, h = x√3

Now, since the triangle formed by the initial position of the car (i.e. ΔABC) is an isosceles triangle, AB = BC


i.e. BC = x√3
∴ DC = x√3 - x = x(√3- 1)
Time taken to travel distance DC is 15 minutes, thus,

Time taken to travel distance x = 
Hence, option 1.

CAT Practice Test - 17 - Question 76

Compound interest on P (P is a 3-digit number) is Rs. 625 for 3 years compounded annually. Which of the following will be the interest on one third of P at twice the rate at simple interest for 11 years?

Detailed Solution for CAT Practice Test - 17 - Question 76

Let the amount be P and rate be r% p.a.
Assume that r = 100.

Then above equation becomes

P + 625 = P x 23 = 8P IP - 625 but then P will not be a 3-digit number.
Thus, r< 100
Compound interest for 3 years:

P( 1 + R)3 - P = PR{R2 + 3R + 3)< 7PR (v R < 1)... (i)

Simple interest on one third the sum for twice the rate for 11 years:
From (i);

C.l. < 7PR < 7.34PR = S.l.
Hence, option 4.

CAT Practice Test - 17 - Question 77

Let the set S consists of numbers 1, 2, 4, 8, 16, and so on till 128. A set K is said to be divisible by another set L if every elements of K can be divided completly by either an element of set L or by the result of addition of any number of elements of set L. Which of the following sets cannot be divided by set S?

i. A = {1, 2, 3......100}

ii. B = {1, 2, 3, 5, 7, 11, ...101}

iii. C = {1, 4, 9, 16, 25...... 100}

Detailed Solution for CAT Practice Test - 17 - Question 77

Set S contains elements which are nothing but powers of 2.
The property with powers of 2 is that we can form any number using 

them by addition.
For instance, 7 can be formed by 1 + 2 + 4 21 can be formed by 1 +4 + 16

41 can be formed by 32 + 8 + 1

49 can be formed by 32 + 16 + 1 and so on.
Thus, all the sets A, B, C would be divisible by adding a combination of elements of set S.
Hence, option 4.

CAT Practice Test - 17 - Question 78

In a grocery shop, customers were asked to stand in a queue in order to buy the goods. It was noticed that the first customer bought 10 kg of rice and then 1/5th of whatever remained in the rice container. The second customer bought 20 kg of rice and then 1/5th of whatever remained. The third customer bought 30 kg of rice plus 1/5th of whatever remained in the container and this went on for each customer in the queue. It was noticed that finally each customer had bought the same amount of rice.

Which of following statements is/are true?

i. Each customer buys 40 kg of rice.

ii. Initially there was 260 kg of rice.

iii. There were exactly 5 customers standing in the queue.

Detailed Solution for CAT Practice Test - 17 - Question 78

Let x be the amount of rice present in the shop initially.
The amount of rice bought by the first customer = 10 + 0.2(x- 10) = 8 + 0.2x
The amount of rice bought by the second customer = 20 + 0.2[x- (20+ 8 + 0.2x)] = 14.4 + 0.16x Now, as each customer buys equal amount of rice, we get, 8 + 0.2x = 14.4 + 0.16x x = 160 Initially there was 160 kg of rice.
Rice bought by each customer = 8 + 0.2x = 40 kg There were exactly 4 customers standing in the queue as they buy 40 kg rice each.
Only statement (i) is correct.
Hence, option 2.

*Answer can only contain numeric values
CAT Practice Test - 17 - Question 79

To make curd, yeast is added to milk. The number of yeast after first second is [log101]. Every nth second the increase in the number of yeast is [log10n] where n is a natural number and [x] denotes the greatest integer less than or equal to x. What is the minimum value of n if after the nth second the number of yeast is 2n?


Detailed Solution for CAT Practice Test - 17 - Question 79

The number of yeast after n seconds = [log101] + [log102] + ... + [log 102] According to the given condition we have, [log10 1] + [log102] + ... + [log 10n] = 2 n

Case 1:

The number of yeast after 1, 2, 3,... 9 seconds is 0. ( log10x< 1 for 1 < n < 9 there fore, [log10x] = 0)

By the given condition, 0 = 2n This case is inadmissible, because n is a natural number.

Case 2: 10 < n < 99

The number of yeast after 10, 11, 12,... 99 seconds is 1. (1 < log10x < 2 for 10 < n < 99 therefore, [log10x] = 1)

(0 + 0 + ...9 times) + (1 +1 + ...(n- 9) times) = 2n n - 9 = 2n n = - 9
This case is also inadmissible, because n is a natural number.

Case 3: 100 < n < 999

The number of yeast after 100, 101, 102,... 999 seconds is 2. (2 < log10x < 3 for 100 < n < 999 there fore, [log10x] = 2)

(0 + 0 + ...9 times) + (1 +1 + ...90 times) + (2 + 2 + ... (n - 99) times) = 2 n

90 + 2/7 - 198 = 2n

90 - 198 = 0
As this is not possible mathematically, therefore, this case is also inadmissible.
Case 4: 1000 < n < 9999

The number of yeast after 1000, 1001, 1002,... 9999 seconds is 3. (3 < log10x < 4 for 1000 < n < 9999,therefore, [log10x] = 3)

(0 + 0 + ...9 times) + (1 +1 + ...90 times) + (2 + 2 + ... 900 times) + (3 + 3 + ... { n - 999) times) = 2n 1890 + 3(n - 999) = 2n

n= 1107

Answer: 1107

CAT Practice Test - 17 - Question 80

A point A (2, - 2) exists in XY plane. It is connected to origin as OA, If OA is elevated to bring the point in 3rd dimension, what shall be the co-ordinates of A'? If OA' is rotated around Z-axis keeping O fixed a circle is formed as shown in the figure. If A' is one end of the diameter (A'D') of the circle what are the co-ordinates of A’ and D’?

Detailed Solution for CAT Practice Test - 17 - Question 80

Let length of OA be d units.
A' = (d x cos 45°, -d x cos 45°, d x sin 45°) By distance formula,
d = 2 √2
A ' = (2, - 2 , 2)

Opposite end of the diameter A'D' would be nothing but mirror image of A' in space with negative X-axis, positive Y-axis and positive Z- axis.

 D = ( - 2 , 2 , 2)
Hence, option 1.

CAT Practice Test - 17 - Question 81

In a certain meteorological model, the temperature of a city (in degrees) is postulated to vary as 20(cos2 θ + sin4θ), where 6 is a parameter that varies such that sin 6 takes on all possible values between -1 and 1 over the course of its variation. What is the minimum possible temperature (in degrees) in this city? 

Detailed Solution for CAT Practice Test - 17 - Question 81

To solve this problem, if f(θ) = 20(cos2 θ + sinθ), we need to find the minimum possible value of f(θ) as the parameter varies over all possible values.
We have 20(cos2θ + sin4θ) = 20(1 - sin2θ + sin4θ) 
Now, we know that, since it is the square of real quantity.
Its minimum value will be zero. This will correspond to the minimum value of the function,
which will equal  = 15 degrees
.

CAT Practice Test - 17 - Question 82

Six different cookies are to be distributed among Subhash and his 10 friends. What is the probability that Subhash gets three cookies?

Detailed Solution for CAT Practice Test - 17 - Question 82

As there are 11 children and 6 cookies, each cookie can be given to any one of the 11 children in 116 ways.
Subhash gets 3 cookies in 6C3 = 20 ways.
So, the remaining 3 different cookies can be distributed among remaining 10 children in 103 = 1000 ways.
∴ Required probability
Hence, option 2.

*Answer can only contain numeric values
CAT Practice Test - 17 - Question 83

A = {1, 4, 7, 10 .... 1999} 
8 = {1, 5, 9, 13 .... 1997} 
C = {1, 6, 11, 16 .... 1996}
Find the number of elements which are present in the union of all the three sets A, B and C.


Detailed Solution for CAT Practice Test - 17 - Question 83

It can be seen that numbers are increasing by 3 in set A, 4 in set B and 5 in set C.
We have to find n (A U B U C). n(A) = [(1999-1)/3] + 1 =667

n (B) = [(1997 - 1)/4] + 1 = 499 + 1 = 500

n (C) = [(1996 - 1)/5] + 1 = 399 + 1 = 400

(A ∩ B) = {1, 13, 25 ... 1993} — This is because 12 is the LCM of 3 and 4.

n (A ∩ B) = [(1993 — 1)/12] + 1 = 167

(B ∩ C) = {1,21,41 ...1981} — This is because 20 is the LCM of 4 and 5.

n(B ∩ C)  = [(1981 - 1 )/20] + 1 = 100

(C ∩ A) = {1, 16, 31 ... 1996} — This is because 15 is the LCM of 5 and 3.

n(C ∩ A)  = [(1996 - 1 )/15] + 1 =133 + 1 =134

(A ∩ B ∩ C) = {1, 61, 121 ... 1981} — This is because 60 is the LCM of 3, 4 and 5.

(A ∩ B ∩ C) = (1981 - 1)/60 = 33 + 1 = 34.
n (A ∪ B ∪ C) = n (A) + n (B) + n(C) - n (A ∩ B) - (B ∩ C) - (C ∩ A) + n (A ∩ B ∩ C)
n (A ∪ B ∪ C) = 667 + 500 + 400 - 167 - 100 - 134 + 34
n(A ∪ B ∪ C) = 1200
Answer: 1200

CAT Practice Test - 17 - Question 84

The revenues and expenditures of companies X and Y are in the ratio 8 : 9 and 14 : 16 respectively. If each of the companies record a profit of Rs. 16.5 lakhs, what is the revenue of X and the expenditure of Y (in Rs. lakhs)?

Detailed Solution for CAT Practice Test - 17 - Question 84

Let the revenue of X and Y (in Rs. lakhs) be 8x and 9x respectively.
Let the expenditure of X and Y (in Rs. lakhs) be 14y and 16yrespectively.

8x - 14y = 9x - 16y.

x = 2y Also, 8x - 14y = 16.5; 16y-14y= 16.5 2y= 16.5
Revenue of X = 8x = 8 * 2y = 8 x 16.5 = Rs. 132 lakhs And, expenditure of Y = 16y = 8 * 2y = 8 x 16.5 = Rs. 132 lakhs

Hence, option 1.

CAT Practice Test - 17 - Question 85

Nathu and Muthu have their shops next to each other but they buy their goods from different wholesalers such that the cost of goods for Nathu is 20% higher than that for Muthu. However, Nathu sells his goods at a price that is 40% higher than the selling price of Muthu, who sells his goods at a price that is 1.5 times his cost price. What is the ratio of the profit percentages of Muthu and Nathu?

Detailed Solution for CAT Practice Test - 17 - Question 85

Let the cost price for Muthu be x.
Cost price to Nathu would be 1.2x. ...[CP for Nathu is 20% higher than Muthu’s CP]

Selling price for Muthu will be 1.5x. ...[SP for Muthu is 1.5 times his CP]

Selling price for Nathu = 1.4 * 1.5x = 2.1x ...[SP for Nathu is 40% higher than Muthu’s SP]
Profit percentage of Muthu = 
Profit percentage of Nathu = 
∴ Ratio of profit percentage = 
Hence, option 3.

CAT Practice Test - 17 - Question 86

Cocktail A contains vodka, whisky and rum in the ratio of 1 : 2 : 3 and cocktail B contains the same in the ratio of 3 : 4 : 5. A cocktail C is made by mixing the two cocktails in a certain ratio. Which of the following is an obtainable ratio of vodka, whisky and rum in the new cocktail? 

Detailed Solution for CAT Practice Test - 17 - Question 86

The ratio of Vodka, Whisky and Rum in the given cocktails is 1 : 2 : 3 and 3 : 4 : 5 .
We can write the given two ratios as 
Amount of Vodka in any mixture of these two cocktails will be,

From the ratios given in the options, only option 2 satisfies the above equation.
Hence option 2.

*Answer can only contain numeric values
CAT Practice Test - 17 - Question 87

Ten cards are numbered 17, 43, 56, 87, 23, 45, 91, 05, 65, and 08. Five of the ten cards are chosen and a number A is formed with the 2-digit numbers on the cards written in some sequence. B is the number formed by the numbers on remaining 5 cards. (For example: If cards numbered 17, 43, 56, 87 and 08 are chosen, A =1743568708 or 1756430887 and so on. Similarly, B is formed with the remaining cards.) If A and B are such that A x B is maximum, find sum of digits of (A - B).


Detailed Solution for CAT Practice Test - 17 - Question 87

For the product to be maximum, A and B should take maximum possible values such that the difference between them is minimum.
For this, let us arrange all the pairs in descending order: 91, 87, 65, 56, 45, 43, 23, 17, 08 and 05.
As we need to have maximum possible values for A and B; first two digits of A and B must be 91 and 87.
Without loss of generality, first two digits of A are 91 and that of B are 87.
Of the remaining, the two highest numbers are 65 and 56.
There are two cases possible.
Case 1: The numbers are 9165 and 8756 Case 2: The numbers are 9156 and 8765 The difference between the numbers in case 1 and case 2 is 409 and 391 respectively. So the product will be maximum in the case 2.
Continuing the same way we form the two numbers as A = 9156431705 and B = 8765452308.
A - B = 9156431705 - 8765452308 = 390979397
Sum of the digits = 56 Answer: 56

CAT Practice Test - 17 - Question 88

Let P be a m digit positive integer such that P3 has n digits. Which of the following cannot be a possible value of m + n?

Detailed Solution for CAT Practice Test - 17 - Question 88

As P has m digits, we can write 10(m-1) < P < 10m
∴ We can further write 10(3m -3) < P3 < 103m
If P3 has n digits, then n can have (3m - 2) digits or (3m - 1) digits or 3m digits.
(m + n) can have (4m - 2) digits or (4m - 1) digits or 4m digits.
From the given options, only 2009 does not satisfy our required condition.
Hence, option 4.

*Answer can only contain numeric values
CAT Practice Test - 17 - Question 89

The set S has elements of the form 2n, with n taking on all natural number values. Let m be any element of S. If p is the number of real roots of the equation xm + 1 - x = 0, how many different values are possible for p?


Detailed Solution for CAT Practice Test - 17 - Question 89

Let m - 2n for some natural number n.
Consider the equation,

xm+1- x = 0

 x(xm - 1) = 0

x = 0 or xm = 1

 m is even,

the possible real roots of the equation are, -1, 0 and 1.
Since p = 3 for all n and m, there is only one possible value of p. Answer: 1

CAT Practice Test - 17 - Question 90

In the quadrilateral ABCD shown below, AED, AEB and BCD are right angles, and AD - 20 cm. BE = x, AE = x + 3, BC = x + 6 and CD = x + 11. What is the value of x?

Detailed Solution for CAT Practice Test - 17 - Question 90

By Pythagoras theorem, we have BC2 + CD2 = BD2 = (x + 6)2 + (x + 11)2
Also DE2 = AD2 - AE2 = 202 - (x + 3)2 Also, BD = BE + DE

Substituting the values of x from the options given into this expression, we find that x = 9 cm satisfies the expression.
Hence, option 2.

CAT Practice Test - 17 - Question 91

The table below shows the age-wise distribution of the employees of TF pvt. ltd. The number of employees aged below 35 years is 432.

Q.If the ratio of females to males in the ‘below 25 years’ age group is 0.86, then what is the number of females in that age group? 

Detailed Solution for CAT Practice Test - 17 - Question 91

Population below 35 years of age = 11 + 23 + 20 = 54 % of the total strength of TF pvt. Ltd.
The total strength of TF 

11 % of the employees are below 25 years of age.
11% of the total strength of TF pvt. ltd. 
The ratio of females (f ) to males (m ) in the ‘below 25 years’ age group is 0.86.

f ~ 41
Hence, option 1.

CAT Practice Test - 17 - Question 92

A small child observes the movement of the minute hand on a large street clock. He sees that the minute hand reaches to the edge of the clock. He wants to find the distance the tip of the minute hand moves in half an hour, and calls this quantity D.

Suddenly, he sees the following inscription at the bottom of the clock tower: The tip of the minute hand of the clock moves at a rate of π/25 m/min. 

Detailed Solution for CAT Practice Test - 17 - Question 92

In an hour, it completes a full circle of the clock or travels a distance of 2D.
∵ The tip of the minute hand moves at a speed of 
It moves at a speed of 


Hence, option 4.

*Answer can only contain numeric values
CAT Practice Test - 17 - Question 93

A and B are two solid iron ball bearings. Surface area of ball bearing A is 300% higher than the surface area of ball bearing B. The volume of ball bearing B is found to be k% lower/higher than the volume of ball bearing A. The value of k must be: (Note: Round off your answer upto one decimal place, if required.)


Detailed Solution for CAT Practice Test - 17 - Question 93

Ratio of areas of two ball bearings =sA : SB = 4 : 1

Ratio of their radii =rA : rB = 2 : 1

Ratio of their volumes = vA: vB = 8 : 1 Volume of B is 12.5% (1/8th) of the volume of B.
But, volume of B is k% less than A. .% k= (100- 12.5) = 87.5%

Answer: 87.5

CAT Practice Test - 17 - Question 94

If y log x + log (log z) = log (xyz) + log [log x + log y + log z], which of the following options is true, given that x, y and z are distinct positive real numbers?

Detailed Solution for CAT Practice Test - 17 - Question 94

y log x + log (log z) = log (xyz) + log [log x + log y + log z]

log xy + log (log z) = log (xyz) + log [log (xyz)]

log (xyx log z) = log [xyz x log (xyz)]

xy x log z = xyz x log (xyz)

zxy = (xyz)xyz

Hence, option 3.

CAT Practice Test - 17 - Question 95

In a planet inhabited by ZUZUS the alphabet used is the same as that in English but in reverse order i.e., letter z comes first, then comes y and so on till the last letter a. If all letters of the word ACHHKM are to be arranged in their dictionary then what will be the 78th word?

Detailed Solution for CAT Practice Test - 17 - Question 95

The first word that can be found in the dictionary is MKHHCA.
Words starting with letter M = 5I/2! = 60 Words starting with letter KM = 4I/2! = 12 Words starting with letter KHM = 3! = 6 KHMACH is the 78th word.
Hence, option 4.

CAT Practice Test - 17 - Question 96

If for real a, b and c, ab + be + ca = 2 then a2 + b2 + c2 cannot take which of the following values?

Detailed Solution for CAT Practice Test - 17 - Question 96

Using the information given in the question in the identity (a + b + c)2

= a2 + b2 + c2 + 2(ab + be + ca), we get

(a + b + c)2 = a2 + b2 + c2 + 4 ...(i)

If a2 + b2 + c2 = 0

But then, ab + be + ca  ≠ 2

So, a2 + b2 + c≠ 0

Also, a2, b2, c2 > 0, therefore, a2 + b2 + c2>0.
So, a2 + b2 + c2 ≠ -1

Hence, option 4.

CAT Practice Test - 17 - Question 97

Rampal started at 3 pm from A for B on a straight road. He covered half the distance at S1 km/hr and then increased his speed to S2 km/hr for the remaining distance. At the same time, Arvind started from A at a speed S2 km/hr and travelled in a straight line to C, then he travelled from C to B in a straight line, at the same speed. D is on the straight road joining A and B. Road joining C and D is perpendicular to that joining A and B, such that AD = BD = CD = 50 km. If both took equal time to reach point B, then find S2/S1.

Detailed Solution for CAT Practice Test - 17 - Question 97

Using Pythagoras theorem
AC ≡ CB ≡ 50√2
Time taken by Rampal to cover the distance along AB

Time taken by Arvind to cover the distance (AC + CB)

As both took equal time to reach B,


Hence, option 1.

CAT Practice Test - 17 - Question 98

Line x is a line parallel to the x-axis. Lines p and q are at angle 70° and 60° respectively w.r.t line x. If p and q are rotated clockwise by a° and b° respectively, such that a = b + 10, what is the magnitude of the minimum possible angle between lines p and q?

Detailed Solution for CAT Practice Test - 17 - Question 98

Since the orientation of the two lines (i.e. clockwise or anti-clockwise) is not given, consider each case.
Assume that lines p and q are both anti-clockwise w.r.t the x-axis, When they are rotated at a° and (a - 10)° clockwise, their final orientation w.r.t the x-axis becomes:

Line p: (70 - a)° Line q: [60 - (a - 10)]° i.e. (70 - a)° In this case, the lines p and q are parallel and the magnitude of the angle between them is 0.
Since the magnitude of the minimum angle is required, this value cannot be less than 0.
Since the minimum angle has been established as 0° in this case, there is no need to check any other case.
Hence, option 2.

CAT Practice Test - 17 - Question 99

Let f(x) = 

Domain of this real function f(x) is R - {5}, where R is the set of all real numbers. What is the range of the function f(x)? 

Detailed Solution for CAT Practice Test - 17 - Question 99

The function can be written as,

y(x - 5) = (x + 3)
xy - 5y = x + 3 x(y - 1) = 5y + 3

y can be any real number except 1, hence the range of the original function f(x) is all real numbers except 1, i.e. R - {1}.
Hence, option 4.

*Answer can only contain numeric values
CAT Practice Test - 17 - Question 100

Mr. Baker runs a bakery shop. One day he made some cakes of different weights. The weight of the heaviest two was 25% of the total weight of all the cakes and the weight of the lightest 5 cakes was 45% the total weight of all cakes. By the end of the day, he was able to sell the 5 lightest cakes. He took the two heaviest cakes home and closed the shop. Next morning he found that all the remaining cakes, which were of same weight, were stolen. How many cakes were stolen?


Detailed Solution for CAT Practice Test - 17 - Question 100

Let the total weight of all the cakes be W.

The weight of the heaviest two cakes = 0.25 W

The average weight of each of the heaviest cakes = 0.25W/2 = 0.125 W
The weight of the lightest five cakes = 0.45W

The average weight of each of the lightest cakes = 0.45W/5 = 0.09W

The weight of the stolen cakes =W-0.25W-0.45W= 0.3W

Let the number of stolen cakes be n. 0.09W < 0.3W/n < 0.125W

The only natural number that satisfies this condition is n = 3.

Answer: 3

2 videos|30 docs|92 tests
Information about CAT Practice Test - 17 Page
In this test you can find the Exam questions for CAT Practice Test - 17 solved & explained in the simplest way possible. Besides giving Questions and answers for CAT Practice Test - 17, EduRev gives you an ample number of Online tests for practice
2 videos|30 docs|92 tests
Download as PDF

How to Prepare for CAT

Read our guide to prepare for CAT which is created by Toppers & the best Teachers
Download free EduRev App
Track your progress, build streaks, highlight & save important lessons and more!
(Scan QR code)